Abdominal Pain

109
Abdominal Pain A 17-year-old boy is taken to the emergency department because he has developed severe abdominal pain. The pain began abruptly several hours previously, and was felt initially in the periumbilical region, but later shifted to the right lower quadrant. The boy had initially felt somewhat nauseous, but this has passed. On physical examination, he is noted to have localized pain on cough and to be running a low-grade fever. Q 1 Examination of the abdomen demonstrates right lower quadrant tenderness at the junction of the middle and outer thirds of the line joining the umbilicus to the anterior superior spine of the iliac. This location is known as which of the following? / A. Gubernaculum / B. Langer's line / C. Linea alba / D. McBurney's point / E. Tunica albuginea Q 2 Which of the following is the most likely diagnosis? / A. Appendicitis / B. Diverticulitis / C. Gallstones / D. Rectal ulcer

Transcript of Abdominal Pain

Page 1: Abdominal Pain

Abdominal Pain

A 17-year-old boy is taken to the emergency department because he has developed

severe abdominal pain. The pain began abruptly several hours previously, and was felt

initially in the periumbilical region, but later shifted to the right lower quadrant. The boy

had initially felt somewhat nauseous, but this has passed. On physical examination, he

is noted to have localized pain on cough and to be running a low-grade fever.

Q 1

Examination of the abdomen demonstrates right lower quadrant tenderness at the

junction of the middle and outer thirds of the line joining the umbilicus to the anterior

superior spine of the iliac. This location is known as which of the following?

/ A. Gubernaculum

/ B. Langer's line

/ C. Linea alba

/ D. McBurney's point

/ E. Tunica albuginea

Q 2

Which of the following is the most likely diagnosis?

/ A. Appendicitis

/ B. Diverticulitis

/ C. Gallstones

/ D. Rectal ulcer

/ E. Renal colic

Page 2: Abdominal Pain

Q 3

The patient also exhibits an increase in pain in the right lower quadrant from the passive

extension of the right hip joint. This finding suggests that the inflammation also involves

which of the following?

/ A. BIadder

/ B. External oblique muscle

/ C. Femur

/ D. IIiopsoas muscle

/ E. Transverse abdominal muscle

Q 4

The patient is prepared for immediate surgery. Cefotaxime is administered before,

during, and after surgery. The specimen, once removed, is sent to the laboratory for

pathology and bacteriologic culture. A malodorous pus surrounds the serosa of the

surgical specimen, and a mixed gram-negative flora is cultured. Rapid enzyme tests for

beta-Iactamase production are positive. Which of the following drugs should be added

to the initial cefotaxime regimen?

/ A. Bacitracin

/ B. CIavulanic acid

/ C. CIindamycin

/ D. Isoniazid

/ E. Vancomycin

Page 3: Abdominal Pain

Q 5

The patient's postoperative recovery is uneventfuI, but 10 days after discharge, he

returns to his physician complaining of continuous low-grade fever. An abscess is

drained transrectally, and organisms are cultured from the pus. Which of the following is

an attribute of this organism that makes it an important abscess former?

/ A. It is an anaerobe

/ B. It is an intracellular pathogen

/ C. Its endotoxin lacks 2,3-ketodeoxyoctonate

/ D. Mycolic acid

/ E. Prodigious capsule

Abdominal Pain Case 5 Answers

A1

The correct answer is D. The point described is McBurney's point, which

overlies the location of the appendix in most individuals.

The gubernaculum (choice A) is the fibrous cord that connects the primordial

testis or ovary to the anterolateral abdominal wall.

Langer's lines (choice B) are the cleavage lines of the skin.

The linea alba (choice C) is a sheet-like aponeurosis that covers the anterior

abdominal wall.

The tunica albuginea (choice E) is a tough fibrous coat that covers the testis.

Page 4: Abdominal Pain

A2

The correct answer is A. This patient has a typical presentation for

appendicitis, and the diagnosis is confirmed by the presence of localized

tenderness at McBurney's point.

Diverticulitis (choice B) is usually a disease of middle-aged or older individuals

and most commonly affects the left-lower quadrant.

Symptomatic gallstone disease (choice C) causes pain and tenderness in the

right upper quadrant.

Rectal ulcer (choice D) causes pain with stool movement, but does not usually

produce tenderness identifiable on abdominal examination.

Renal colic (choice E) usually produces flank or lower back pain.

A3

The correct answer is D. This patient has a "positive psoas sign," which is an

increase in pain from passive extension of the right hip joint. This maneuver

stretches the iliopsoas muscle, which lies behind the appendix and can become

secondarily inflamed when the appendiceal inflammation extends through the

serosa. The psoas sign is clinically useful in both confirming the appendix as the

probable origin of the patient's pain, and indicating that the inflammation is

transmural and that the risk of rupture and peritonitis is increased.

The bladder (choice A) is located more medially, and is usually not affected by

appendicitis.

The external oblique (choice B) and transverse abdominal (choice E) muscles

are in the anterior and lateral abdominal walls, and do not usually become

inflamed with appendicitis.

The femur (choice C) is moved during the extension of the right hip joint, but is not the source of

the pain.

Page 5: Abdominal Pain

A4

The correct answer is B. Clavulanic acid is a beta-lactamase inhibitor, which

when administered with beta lactam agents, irreversibly binds and inactivates

bacterial beta-lactamases, thereby permitting the companion drug to disrupt

bacterial cell wall synthesis. Suspected appendicitis is usually treated with

prompt appendectomy, since delay is associated with increased risk of

potentially life-threatening peritonitis and sepsis.

Bacitracin (choice A) is not correct, since this drug inhibits bacterial cell wall

synthesis by binding to and inhibiting the dephosphorylation of a membrane-

bound lipid pyrophosphate. Gram-negative bacteria are resistant to this agent,

and it would not have a synergistic effect if administered with a third generation

cephalosporin.

Clindamycin (choice C) is not correct, because this drug blocks protein

elongation by binding to the 50S ribosome. Although it is effective against

anaerobic gram-negative bacilli, it would not have a complementary effect when

administered with a third generation cephalosporin.

Isoniazid (choice D) is not correct because it inhibits the synthesis of mycolic

acids for the cell wall of actively dividing Mycobacteria. It would not be effective

in the flora of this patient's gut, nor would it act synergistically with third

generation cephalosporins.

Vancomycin (choice E) is not correct because it disrupts cell wall synthesis in growing gram-

positive bacteria. It would not be effective against the flora of this patient's gut, nor would it act

synergistically with third generation cephalosporins.

Page 6: Abdominal Pain

A5

The correct answer is E. Prevotella (Bacteroides) is a frequent cause of

abscesses in the intestinal tract because it is a normal flora organism and

produces a large capsule, which impedes phagocytosis.

Although the genus is anaerobic (choice A), it is not this attribute which causes

its formation of abscesses.

Prevotella is extracellular, not an intracellular pathogen (choice B).

Although Prevotella does indeed have this type of endotoxin (choice C), the

absence of this molecule decreases the toxicity of the toxin, and does not

contribute to its proclivity toward abscess formation.

Mycobacteria, and not other genera such as Prevotella, are known for their long-chain fatty acids

(mycolic acids; choice D).

Abdominal Pain Case 4

A 45-year-old man goes to an emergency department because he is experiencing

severe abdominal pain, which is radiating straight through to his back. The pain began

several hours after an admitted alcoholic binge, and has not changed in position,

although it has become worse.

Q 1

Which of the following would be the most likely cause of this type of pain?

/ A. Acute appendicitis

/ B. Acute hepatitis

/ C. Acute pancreatitis

/ D. Chronic hepatitis

Page 7: Abdominal Pain

/ E. Myocardial infarction

Q 2

In addition to alcohol use, which of the following is a common predisposing factor for

this patient's disease?

/ A. Biliary tract stones

/ B. Duodenal cancer

/ C. Gastric carcinoma

/ D. Kidney stones

/ E. Peptic ulcer

Q 3

Marked serum elevation of which of the following markers would most strongly

substantiate the likely diagnosis?

/ A. Acid phosphatase

/ B. Amylase

/ C. Aspartate aminotransferase

/ D. AIkaline phosphatase

/ E. Creatinine kinase

Page 8: Abdominal Pain

Q 4

The patient has a severe course that requires treatment in an ICU. CIinically, he

appears similar to patients with sepsis, with fever, elevated white count, hypotension,

increased pulse rate, shallow and rapid breathing, oliguria, and a blunted sensorium, in

addition to his pain and abdominal tenderness. These clinical findings are most likely

related to which of the following?

/ A. Activation of the inflammatory cascade

/ B. AIcohol withdrawal symptoms

/ C. AIIergic reaction to alcohol

/ D. Drug toxicity effect

/ E. Secondary infection with mixed flora gut bacteria

Q 5

The patient's condition resolves in about two weeks, but he continues to drink after

leaving the hospitaI. When seen several years later, he has had a number of similar

episodes, and now has chronic severe abdominal pain. CT scan demonstrates a single,

smooth-walled, fluid filled space in the tail of the pancreas, which can be reached by the

radiologist for CT-guided aspiration with an approach from the back. The fluid aspirated

is yellowish, clear, and acellular. Which of the following is the most likely diagnosis?

/ A. Pancreatic microcystic adenoma

/ B. Pancreatic mucinous cystadenocarcinoma

/ C. Pancreatic mucinous cystadenoma

/ D. Pancreatic pseudocyst

/ E. Pancreatic solid-cystic tumor

Page 9: Abdominal Pain

____________________________________________________________________

Abdominal Pain Case 4 Answers

A1

The correct answer is C. The typical pain described occurs in approximately

50% of patients with acute pancreatitis. Other patients may have milder pain or

even, uncommonly, pain first felt in the lower abdomen.

The pain of acute appendicitis (choice A) is often felt first as referred pain near

the umbilicus, with tenderness on palpation in the left lower quadrant.

Acute hepatitis (choice B) can cause pain referred to the right shoulder.

Chronic hepatitis (choice D) does not usually cause pain.

Myocardial infarction (choice E) can cause substernal pain and pain radiating to the left

shoulder.

A2

The correct answer is A. The overwhelmingly most common predisposing

factors for acute pancreatitis are gallstones (more specifically tiny ones that

lodge in the extrahepatic bile duct system) and alcohol abuse.

Rarely, nearby cancers (choices B and C) can occlude the pancreatic duct

system and cause a secondary acute pancreatitis.

Kidney stones (choice D) have no relationship with pancreatitis.

Peptic ulcers (choice E) that erode into the pancreas can uncommonly secondarily inflame the

pancreas

Page 10: Abdominal Pain

A3

The correct answer is B. The usual markers for pancreatitis are amylase and

lipase. Marked elevation of amylase usually means either pancreatic disease or

salivary gland disease; lipase will be elevated in pancreatic disease but not

salivary gland disease. If you see elevated amylase on a USMLE question, you

should think of pancreatitis or salivary gland disease (mumps, salivary gland

stone). However, you should be aware, for your general medical knowledge, that

modest elevations of amylase can be seen in a much wider variety of settings

(often reflecting either subclinical pancreatic damage or hemoconcentration of

pancreatic enzymes), including GI obstruction, mesenteric thrombosis and

infarction, macroamylasemia (a genetic condition with abnormal amylase), renal

disease, ruptured tubal pregnancy, lung cancer, acute alcohol ingestion, and

following abdominal surgery.

Associate acid phosphatase (choice A) with diseases involving the prostate

and, to lesser degrees, bone, the heart, platelets, and the liver.

Associate aspartate aminotransferase (choice C) with diseases of the heart,

muscle, liver, pancreas (though not as important for diagnosis as amylase and

lipase), and brain.

Associate alkaline phosphatase (choice D) with diseases of bone, liver, and to

lesser degrees, lung and heart.

Associate creatinine kinase (choice E) with diseases of the heart, muscle, brain, and the general

body (trauma, surgery).

A4

The correct answer is A. Acute pancreatitis can either be relatively mild, or a

severe condition that may cause death. It is thought that, in severe cases,

leakage of enzyme-containing pancreatic secretions into the tissues/and or

blood stream causes cleavage of precursors, thus strongly activating the

complement and inflammatory cascades. These, in turn, produce abundant

Page 11: Abdominal Pain

cytokines, which worsen the symptoms. The clinical result is similar to sepsis,

with risk of multi-organ failure and death. The treatment of acute pancreatitis is

primarily supportive, and may include careful attention to fluid resuscitation,

oxygen supplementation, cardiovascular support, dialysis, management of

electrolyte abnormalities, pain control, and total parenteral nutrition.

Alcohol allergy (choice C) or withdrawal (choice B) do not play any additional

part in most of these symptoms once the pancreatitis has developed.

Infection (choice E) and drug toxicity (choice D) are also not a necessary part of the clinical

picture, although physicians may worry that the patient's general clinical status is masking other,

potentially more treatable, problems.

A5

The correct answer is D. Pancreatic pseudocyst is a fairly common

complication of both acute and chronic pancreatitis, and appears to develop

when trapping of pancreatic digestive juices (containing amylase, lipase, and

proteases) causes a "digestion" of part of the pancreas, leaving a fluid filled

cystic space. The term "pseudocyst", rather than "cyst", is used by purists

because the space does not have an epithelial lining, and is hence not a "true

cyst". Pseudocysts are usually solitary and typically measure 5-10 cm in

diameter. They can be surgically excised (and the surrounding tissue will

typically show evidence of chronic pancreatitis in long-standing cases) or

sometimes, if the anatomy is favorable, drained into adjacent hollow viscera.

Some are medically managed if small.

Most true neoplasms of the pancreas contain (often large numbers of) smaller, multiple, cysts.

These tumors can be benign or malignant, and the ones with mucus-secreting epithelium

(choices B and C) are more common than those with a serous lining (choices A and E).

Abdominal Pain Case 3

A 64-year-old man with a history of coronary artery disease (CAD) comes to the

emergency department with the acute onset of severe, constant, Lower abdominal pain and

rectal bleeding. He reports that he previously has had several episodes of similar, but less severe

pain.

Page 12: Abdominal Pain

About 12 hours after the onset of pain, the patient began passing copious bright red

blood per rectum. He denies nausea, vomiting, sick contacts, or foreign traveI. Initial

physical examination reveals a distressed man, who is afebrile, but tachypneic, with

scant diffuse abdominal tenderness to palpation. Rectal examination is positive for

blood. Laboratory studies reveal a metabolic acidosis with an elevated serum Iactate.

Q 1

Which of the following is the most likely diagnosis?

/ A. Colon carcinoma

/ B. Infectious colitis

/ C. Inflammatory bowel disease

/ D. Ischemic colitis

/ E. Necrotizing enterocolitis

Q 2

The lactate produced from the anaerobic metabolism in the infarcted gut will likely be

which of the following?

/ A. Exhaled as a fruity odor

/ B. Incorporated into glycogen in the liver

/ C. Incorporated into myoglobin in muscle

/ D. Incorporated into urea in the urine

/ E. Secreted by the kidneys unchanged

Page 13: Abdominal Pain

Q 3

If this patient's disease were drug-induced, which of the following agents would most

likely be responsible?

/ A. Acetaminophen

/ B. Amiodarone

/ C. Cocaine

/ D. Dexamethasone

/ E. Nitroglycerin

Q 4

While the patient is in the emergency department, the pain becomes increasingly

severe. Several hours after his initial examination, the patient becomes febrile and is

now exquisitely tender to palpation. He writhes in pain when the physician jostles the

bed. Air is seen under the diaphragm in an upright chest x-ray film. These new findings

suggest which of the following?

/ A. Abdominal aortic aneurysm

/ B. Bowel obstruction

/ C. Cholecystitis

Page 14: Abdominal Pain

/ D. Hypovolemia

/ E. Perforation with peritonitis

Q 5

Upon surgical exploration of the abdomen, the colon is dull and dusky from the mid

transverse colon to the rectum. The patient has occluded

which of the following vessels?

/ A. Celiac trunk

/ B. Cystic artery

/ C. External iliac artery

/ D. Inferior mesenteric artery

/ E. Superior mesenteric artery

Abdominal Pain Case 3 Answers

A1

The correct answer is D. A patient with severe abdominal pain and rectal

bleeding with an unremarkable physical examination is likely suffering from

ischemic colitis. "Pain out-of-proportion to examination" is a classic finding for

ischemic colitis. The previous episodes of less severe pain represent ischemic

angina. An infarction has occurred, as indicated by the rise in serum lactate

secondary to the colon's anaerobic metabolism. The history of coronary artery

disease also suggests this diagnosis, as the atherosclerotic processes that

contribute to his CAD are also likely present in his abdominal vasculature.

Page 15: Abdominal Pain

Colon cancer (choice A) would produce less acute symptoms, but occasionally,

colon cancer may present acutely with obstructive symptoms. Patients may

have bleeding and abdominal pain, but the pain is typically intermittent and

accompanied by nausea, vomiting, abdominal distention, and absence of flatus.

Infectious colitis (choice B) is incorrect. While patients may have bleeding and

abdominal pain, nothing in the history suggests a disease of infectious origin (no

sick contacts or foreign travel). The acute onset also suggests a vascular event,

rather than an infectious one.

Inflammatory bowel disease (IBD) (choice C) is incorrect because while the

patient reports previous episodes, an elderly man with IBD would likely have a

chronic history of abdominal pain and bleeding.

Necrotizing enterocolitis (choice E) affects premature infants and would not be relevant in this

setting.

A2

The correct answer is B. Lactate is converted into glucose, and then glycogen

in the liver by a process know as the Cori cycle.

Choice A is incorrect, as lactate would not be exhaled. A fruity odor on the

breath would be a sign of ketoacidosis.

While some of the carbon from the lactate may be incorporated into peptides

via Krebs intermediates (e.g., choice C), the vast majority would be left as

carbohydrate.

Urea (choice D) represents a means of eliminating nitrogenous waste.

Choice E is wrong, as the kidneys would retain the lactate, rather than excreting it.

A3

The correct answer is C. Cocaine is a sympathomimetic drug that indirectly

acts on both the alpha and beta adrenergic receptors on the vasculature. As

such, cocaine may cause vasospasm in the abdominal vasculature leading to

Page 16: Abdominal Pain

infarction and ischemic colitis. Similar vasospastic events may occur in the

coronary vasculature, leading to myocardial infarction.

Acetaminophen (choice A) is an analgesic, and would not play a role in

producing ischemic colitis.

Amiodarone (choice B) is an antiarrhythmic, and would not contribute to

ischemic colitis.

Dexamethasone (choice D) is a steroidal anti-inflammatory drug. Not only

would this medication not cause ischemic colitis, it might mask the symptoms

due to its potent anti-inflammatory properties.

Nitroglycerin (choice E) is a venodilator, and would not contribute to ischemic colitis. As a

venodilator, nitroglycerin is used to treat coronary ischemia by reducing cardiac preload.

A4

The correct answer is E. This patient has experienced a bowel perforation. Air

under the diaphragm in an upright chest film provides definitive evidence that a

hollow viscus has ruptured. Air near the liver on a left lateral decubitus (patient

lays with the left side down) is an alternative study to demonstrate perforation.

Spillage from the perforated bowel has irritated and inflamed the peritoneum,

resulting in peritonitis. Symptoms of peritonitis include extreme, sharp pain

exacerbated by jostling (patients often report that the bumpy ride to the

emergency department caused extreme pain). Patients will be exquisitely

tender to palpation and percussion and may have abdominal rigidity. Fever

typically accompanies peritonitis.

While an abdominal aortic aneurysm or AAA (choice A) presents as acute

abdominal pain, this pain is described as tearing and may radiate to the back. A

pulsatile abdominal mass may be palpated. The air on the chest film is also

inconsistent with AAA.

This patient does not have bowel obstruction (choice B). Signs and symptoms

of bowel obstruction include: nausea, vomiting, intermittent abdominal pain,

Page 17: Abdominal Pain

hypovolemia, abdominal distention, absence of flatus, and a "step ladder" bowel

pattern on abdominal films.

Cholecystitis (choice C) typically presents as right upper quadrant (RUQ) pain,

fever, and jaundice. Patients usually have a history of colicky RUQ pain.

While the patient is at risk for hypovolemia (choice D), none of the symptoms listed typify

hypovolemia. Signs and symptoms of mild to moderate hypovolemia include malaise, dry

mouth, thirst, decreased skin turgor, tachycardia, hypotension, and decreased urine output.

A5

The correct answer is D. The inferior mesenteric artery distributes blood to

the embryologic hindgut. This includes the distal 1/3 of the transverse colon to

the rectum. The rectum is spared because it receives circulation from the

inferior rectal artery (not mesenteric).

The celiac trunk (choice A) supplies the embryologic foregut. The first three

branches include the splenic artery, the left gastric artery, and the common

hepatic artery. This patient has no findings in this distribution.

The cystic artery (choice B) supplies the gall bladder. There are no gall bladder

findings in this case.

The external iliac artery (choice C) gives rise to the vessels of the lower

extremity. Symptoms of occlusion or stenosis might include buttock and thigh

pain exacerbated by walking. Severe stenosis might give patients buttock and

thigh pain, even at rest.

The superior mesenteric artery (choice E) supplies the embryologic hindgut. This extends from

the duodenum to the proximal 2/3 of the transverse colon

Abdominal Pain Case 2

A 47-year-old woman presents to the emergency department with cra mping/colicky

abdominal pain. The current episode of pain began several hours ago, following a fatty

meaI. The pain began slowly, and rose in intensity to a plateau over the course of

several hours. The patient reports that she had had several other episodes of similar

Page 18: Abdominal Pain

pain during the past several months, with long intervening periods of freedom from pain.

On physical examination, she is noted to have tenderness to deep palpation in the right

upper quadrant of the abdomen near the rib cage. The patient also reports that she is

experiencing shoulder/back pain at a site she identifies near the right lower scapula, but

no tenderness can be elicited during the back and shoulder examination.

Q 1 Which of the following organs is the most likely source of this woman's pain?

/ A. Appendix

/ B. Diaphragm

/ C. Esophagus

/ D. Gallbladder

/ E. Stomach

Q 2

Which of the following techniques would be most appropriate to demonstrate the

patient's most likely diagnosis?

/ A. Colonoscopy

/ B. CT scan of the abdomen

/ C. Esophagoduodenoscopy

/ D. MRI scan of the abdomen

/ E. UItrasonography

Q 3

Following appropriate diagnostic studies, the patient is taken to the surgical suite.

During the surgery, the surgeon inserts his fingers from

Page 19: Abdominal Pain

right to left behind the hepatoduodenal ligament. As he does so, his fingers enter which

of the following?

/ A. Ampulla of Vater

/ B. Common bile duct

/ C. Epiploic foramen

/ D. Greater peritoneal sac

/ E. Portal vein

Q 4

During the cholecystectomy, the surgeon ligates the cystic artery. This is typically a

branch of which of the following?

/ A. Gastroduodenal artery

/ B. Left gastroepiploic artery

/ C. Right gastroepiploic artery

/ D. Right hepatic artery

/ E. Superior pancreaticoduodenal artery

Q 5

Page 20: Abdominal Pain

Pathologic examination of the specimen removed by the surgeon demonstrates the

presence of numerous yellow stones (shown above).

These are most likely composed primarily of which of the following?

/ A. Bilirubinate

/ B. Calcium phosphate

/ C. Cholesterol

/ D. Cystine

/ E. Struvite

Q 6

If this patient had a small stone lodge near the ampulla of Vater, which of the following

complications would be most likely to occur?

/ A. Crohn disease

Page 21: Abdominal Pain

/ B. Diabetes mellitus

/ C. Pancreatitis

/ D. Peptic ulcer

/ E. Polyarteritis nodosa

Q 7

If this patient had refused surgical treatment, which of the following would be the most

appropriate pharmacotherapy to provide definitive

treatment and thereby relieve associated pain?

/ A. Ampicillin

/ B. CIofibrate

/ C. Meperidine

/ D. Oxycodone

/ E. Ursodiol

Abdominal pain Case 2 Answers

A1

The correct answer is D. This woman most likely has gallstones.

Cholelithiasis, or the formation of calculi (gallstones) within the gallbladder, is

very common in the United States, with over 500,000 cholecystectomies being

performed yearly. While many cases of gallstone disease are symptomatic, right

upper quadrant pain with referral of the pain to the lower right scapula should

specifically suggest gallbladder disease. The pattern of episodes of several

hours of pain followed by long periods of freedom from pain is also typical of

symptomatic gallstone disease.

The appendix (choice A) would most likely cause lower abdominal pain.

Page 22: Abdominal Pain

Pain from irritation of the diaphragm (choice B) can cause right upper quadrant

pain and referred pain in the supraclavicular area (rather than the subscapular

pain of biliary colic). The absence of right upper quadrant tenderness to

palpation, and the history of pain after a fatty meal also argue against this

diagnosis.

Esophageal pain (choice C) related to regurgitation of gastric contents

(heartburn) can occur postprandially, but tends to radiate into the neck, throat,

or even face.

Peptic ulcer pain of gastric origin (choice E) is usually described as causing burning, gnawing,

or hunger, and may be relieved by eating.

A2

The correct answer is E. Real-time ultrasonography, with 98% sensitivity and

95% specificity, is considered the method of choice for diagnosing possible

gallbladder stones.

Colonoscopy (choice A) and esophagoduodenoscopy (choice C) might be

helpful for excluding alternative diagnoses, but would not themselves establish

a diagnosis of gallstone disease.

CT (choice B) and MRI (choice D) scans of the abdomen are expensive tests whose use is not

warranted, since real-time ultrasonography performs as well or better.

A3

Page 23: Abdominal Pain

The correct answer is C. The space behind the stomach, hepatoduodenal

ligament, and hepatogastric ligament is the omental bursa. This space can be

entered by passing through the epiploic foramen of Winslow, as described in the

question stem.

The common bile duct enters the duodenum through the ampulla of Vater

(choice A).

The hepatoduodenal ligament contains the common bile duct (choice B), the

portal vein (choice E), and the hepatic artery.

The greater peritoneal sac (choice D) lies anterior to the stomach and hepatoduodenal ligament.

A4

The correct answer is D. The cystic artery is generally a branch of the right

hepatic artery.

The gastroduodenal artery (choice A) is a branch of the (common) hepatic

artery.

The left gastroepiploic artery (choice B) is a branch of the splenic artery.

The right gastroepiploic artery (choice C) is a branch of the gastroduodenal

artery.

The superior pancreaticoduodenal artery (choice E) is a branch of the gastroduodenal artery.

A5

The correct answer is C. The stones are gallstones, and their yellow color

indicates that they are composed of cholesterol. Cholesterol stones are the most

common form of gallstones. Risk factors include female sex, multiparity, obesity,

increased age (female, fat, forty, and fertile) and North American Indian race.

Page 24: Abdominal Pain

Bilirubinate (choice A) gallstones, which are usually associated with hemolytic

anemias, are less common, brown, rather than yellow, and often faceted.

Calcium phosphate (choice B), cystine (choice D), and struvite (choice E) composition can be

seen in kidney stones

A6

The correct answer is C. A small gallstone obstructing the pancreatic outflow is a well-known cause of acute pancreatitis. The other conditions listed are not caused by gallstones

A7

The correct answer is E. The question is asking, "Which of the following will

eradicate a gallstone?" When a gallstone is eliminated the pain will

subsequently be eliminated. This question is NOT asking, "which of the following

is the most appropriate form of pain control?". Ursodiol (ursodeoxycholic acid) is

a hydrophilic bile acid that is used to dissolve small (<>

Analgesics and antibiotics, such as ampicillin (choice A), are administered

when appropriate, but do not help eradicate the stones.

Clofibrate (choice B) is an antihyperlipidemic that is associated with the

development of gallstones. High-risk patients, such as diabetics and the elderly,

should be watched closely.

As a side note, if this question were asking: "which of the following is the most appropriate form

of pain control in this patient", the most appropriate answer would be meperidine. Meperidine

(choice C) is the narcotic of choice since it causes the least amount of spasm of the sphincter of

Oddi. In other words, meperidine is preferred over oxycodone (choice D).

A 65-year-old man presents to his physician because he has been having increasing difficulty swallowing over the past 2 months. He is still able to swallow liquids, but swallowing solid food now causes severe pain and a sense of fullness behind his sternum. He has lost 18 poundssince his swallowing difficulties began. The patient is referred to a gastroenterologist, who demonstrates a mass lesion of the distal esophagus, which on biopsy is shown to contain cancer.

Page 25: Abdominal Pain

Q1Which of the following is most important in separating the esophagus from the larynx, and must consequently be carefully passed behindduring endoscopy?/ A. Arytenoids/ B. Cricoid cartilage/ C. Epiglottis/ D. Pharynx/ E. Vocal cords

Q2Which of the following nerves provides the efferent impulses necessary for the esophageal actions that occur during swallowing?/ A. Glossopharyngeal/ B. Hypoglossal/ C. Spinal accessory/ D. Trigeminal/ E. Vagus

Q3Which of the following approximately represents the proportion of different esophageal cancer types now being observed in the United States?/ A. 1/10 adenocarcinoma and 9/10 squamous cell carcinoma/ B. 1/3 adenocarcinoma and 2/3 squamous cell carcinoma/ C. 1/2 adenocarcinoma and 1/2 squamous cell carcinoma/ D. 2/3 adenocarcinoma and 1/3 squamous cell carcinoma/ E. 9/10 adenocarcinoma and 1/10 squamous cell carcinoma

Q 4Precancerous metaplasia of the esophageal epithelium gives rise to a mucosa resembling which of the following?/ A. Mesothelium/ B. Respiratory epithelium/ C. Small intestine/ D. Squamous epithelium/ E. Stomach

Page 26: Abdominal Pain

Q5Frequent use of which of the following has recently been found to probably have a protective effect against development of esophagealcancer?/ A. Acetaminophen/ B. Alcohol/ C. Aspirin/ D. Cigarettes/ E. Codeine

Q 6Currently, esophageal cancer has which of the following long-term survival rates?/ A. Less than 5%/ B. 30%/ C. 50%/ D. 70%/ E. More than 95%

Dysphagia case 2 answersA1The correct answer is C. Endoscopists are very careful when guiding the endoscope past the epiglottis, which is a pear-shaped portion of elastic cartilage that can be moved during swallowing to close the larynx, preventing swallowed material from eventually entering the lungs.The arytenoids (choice A) are the site of the attachment of the vocal cords (choice E) within the larynx.The cricoid cartilage (choice B) is in the more distal portion of the larynx.The pharynx (choice D) is shared by the respiratory and gastrointestinal tracts.

A2The correct answer is E. The vagus nerve supplies the efferent input into the esophagus that is necessary for swallowing.The glossopharyngeal nerve (choice A) provides taste and sensation on the palate, but the only muscle it supplies is the stylopharyngeus.The hypoglossal nerve (choice B) moves the tongue during the initiation of swallowing, but does not innervate the esophagus.

Page 27: Abdominal Pain

The spinal accessory nerve (choice C) plays no role in swallowing. This nerve mediates head and shoulder movement and innervates laryngeal muscles.The trigeminal nerve (choice D) provides general sensation to the mouth and motor innervation to the muscles of mastication.

A3The correct answer is C. More recent statistics indicate that the incidence of adenocarcinoma and squamous cell carcinoma of the esophagus are now roughly equal. Formerly, approximately 2/3 of the esophageal cancers were squamous in origin (choice B). Adenocarcinoma of the esophagus is often found in the distal esophagus.

A4The correct answer is C. This is an indirect question about Barrett's esophagus, which is an important precursor of adenocarcinoma of the esophagus. While Barrett's esophagus was initially defined to be either gastric-type or intestinal-type metaplasia of the esophagus, more recent studies have shown that the actual problem lesion is more likely to be intestinal metaplasia (diagnosed when isolated goblet cells are seen in the epithelium) rather than gastric metaplasia (choice E).Metaplasia to mesothelium (choice A) or ciliated respiratory epithelium (choice B) does not usually occur in the esophagus.The normal epithelium of most of the esophagus is squamous (choice D).

A5The correct answer is C. An interesting new research observation that may be exploited in the future is that the incidence of esophageal cancer appears to be much lower in people who use aspirin frequently.Cigarettes (choice D) and alcohol use (choice B) have been implicated as risk factors for esophageal cancer.Acetaminophen (choice A) and codeine (choice E) have no known effects on the incidence of esophageal cancer.

A6The correct answer is A. Esophageal cancer is one of the very bad cancers, presently with poor long-term survival. The underlying problem is that the esophagus is only about 3 mm thick, and both metastatic disease and direct spread (often unresectable) to mediastinal structures is common. Active

Page 28: Abdominal Pain

research is presently being undertaken to modify this prognosis by using chemotherapy and radiation therapy prior to surgery, but these modalities have not yet come into widespread use.

A 35-year-old woman consults a physician because she has been having trouble

swallowing. She also often experiences chronic heartburn. The physician

performs a screening physical examination, and notices that the skin of her

hands appears tight and shiny. On specific

questioning, she reports having often experienced color changes in her hands

from white to blue to red.

Q1

Which of the following is the most likely cause of the patient's difficulties with

swallowing?

/ A. Achalasia

/ B. Adenocarcinoma

/ C. Chagas disease

/ D. Scleroderma

/ E. Squamous carcinoma

Q2

The color changes described on the patient's hand are most likely due to which

of the following?

/ A. Arteriolar spasm

/ B. Blood clots at sites of vascular injury

/ C. Large artery spasm

/ D. Platelet clots

/ E. Stasis blood clots

Q 3

Additional findings on physical examination include noting that the skin changes

are limited to areas distal to the elbow and knee, the presence of calcified

Page 29: Abdominal Pain

nodules on the extensor surfaces of the forearms, and the presence of

telangiectasias on the forearms. This suggests

that this patient has which of the following?

/ A. Bauer syndrome

/ B. Charcot syndrome

/ C. CREST syndrome

/ D. Crigler-Najjar syndrome

/ E. Dandy-Walker syndrome

Q 4

If this woman's involved skin were biopsied, which of the following would most

likely be seen?

/ A. Cleft separating the dermis and subcutaneous tissues

/ B. Epithelial cell hyperplasia

/ C. Marked dermal fibrosis

/ D. Narrowing of the basal lamina of small capillaries

/ E. Thickening of rete pegs

Q 5

More than 90% of the patients with the limited cutaneous form of this disorder

make which of the following autoantibodies?

/ A. Anti-centromere

/ B. Anti-DNA topoisomerase l

/ C. Anti-double-stranded DNA

/ D. Anti-Golgi

/ E. Anti-ScI-70

_____________________________________________________________________

Dysphagia case 4 answers

A1

The correct answer is D. The tip-off is the reference to the patient's skin changes

Page 30: Abdominal Pain

that are typical for scleroderma, also known as systemic sclerosis. Scleroderma

is a disease that may be either predominately limited to the skin or involve

many body systems, including the musculoskeletal system, gastrointestinal tract

(with esophageal involvement most often symptomatic), cardiorespiratory

system, and renal system. Esophageal dysfunction is a common complication of

scleroderma. In most patients, the skin changes are obvious, even if the patient

has not been previously diagnosed. Rarely, the skin changes may be noticed at

an earlier stage, in which the skin of the hands appears puffy and edematous,

but not scarred.

Achalasia (choice A) and Chagas disease (choice C) are also important causes of

dysphagia, but in achalasia, the physical examination is usually normal, and in

Chagas disease, you should be able to elicit a history of possible exposure in

Central or South America.

Reflux esophagitis with risk of progression to Barrett esophagus and

adenocarcinoma (choice B) can occur in scleroderma, but it is unlikely, at this

early stage in the patient's disease, that she already has adenocarcinoma.

The risk of squamous cell carcinoma (choice E) is not increased in this patient.

A2

The correct answer is A. The color changes described are typical for Raynaud's

phenomenon, which occurs because of changes in perfusion due to arteriolar

spasm. Raynaud's phenomenon is common in scleroderma, largely because the

subintimal hyperplasia of small vessels characteristic of scleroderma can reduce

the luminal diameter by more than 75%. Some authors argue that the vascular

changes seen in scleroderma are actually the insult that triggers the subsequent

development of fibrosis.

Blood clots at sites of vascular injury (choice B) are typical in clots that form in

arteries on surfaces such as a fractured atherosclerotic clot.

Larger artery spasm (choice C) of vessels damaged by atherosclerosis is thought

to contribute to some myocardial infarctions, but is not thought to be an

important pathologic mechanism in scleroderma.

Platelet clots (choice D) can be seen in some diseases in which platelet function

is abnormal, such as essential thrombocytopenia.

Stasis blood clots (choice E) commonly are found in venous thromboses.

A3

Page 31: Abdominal Pain

The correct answer is C. These findings, together with esophageal dysfunction

and Raynaud's phenomenon (both of which this patient has), are called the

CREST syndrome, also known as limited cutaneous scleroderma. This form of

scleroderma has a better long-term prognosis than when the skin changes also

involve the trunk (diffuse scleroderma) and more internal organs are

additionally involved.

Bauer syndrome (choice A) is aortitis and aortic endocarditis as a complication

of rheumatoid arthritis.

Charcot syndrome (choice B) is intermittent claudication.

Crigler-Najjar syndrome (choice D) is a severe familial liver disease.

Dandy-Walker syndrome (choice E) is a malformation of the central nervous

syndrome.

A4

The correct answer is C. In scleroderma, early changes (at the point at which the

hands appear swollen, rather than with tight, thick skin) show edema with

perivascular infiltrates of CD4+ T cells. At this stage, the collagen fibers are

swollen and beginning to degenerate. The smaller vessels may show basal

lamina thickening (not narrowing as in choice D) and endothelial (not epithelial

cell hyperplasia) cell damage and proliferation. With time, the characteristic

marked dermal fibrosis develops, which tends to both narrow (not thicken as in

choice E) the rete pegs and attach the dermis tightly (compare with cleft

formation as in choice A) to subcutaneous tissues.

A5

The correct answer is A. All forms of scleroderma are thought to have a strong

autoimmune component, and glucocorticoids and azathioprine are used to

suppress the inflammatory complications of scleroderma. (Other drugs that can

be used in therapy include penicillamine, which inhibits collagen cross-linking,

NSAIDS for pain, and ACE inhibitors to protect the kidney if hypertension or renal

damage occurs.) The anti-centromere antibody is quite specific for CREST

syndrome (96% of cases), and is only seen in a minority of patients with diffuse

scleroderma (mainly those with Raynaud's phenomenon) and rarely in systemic

lupus erythematosus and mixed connective tissue disease.

Anti-DNA topoisomerase I (choice B), also called anti-Scl-70 (choice E) occurs

commonly (64-75%) in diffuse scleroderma, but only rarely in CREST syndrome.

Page 32: Abdominal Pain

Anti-double-stranded DNA (choice C) is fairly specific for systemic lupus

erythematosus, although it only occurs in 50-60% of lupus cases.

Anti-Golgi antibodies (choice D) are seen most often in systemic lupus

erythematosus and Sjögren syndrome.

A 30-year-old man consults a physician because he has been having increasing

difficulty swallowing both solids and liquids. Physical examination of the patient

is noncontributory. Barium swallow studies show a mostly dilated esophagus

with slow passage of barium into the

stomach. The very distal part of the esophagus appears narrowed into a "bird's

beak." Esophageal manometry shows incomplete relaxation of the lower

esophageal sphincter in response to swallowing, high resting lower esophageal

pressure, and absent esophageal peristalsis.

Q 1

The manometry and barium swallow studies most strongly support which of the

following diagnoses?

/ A. Achalasia

/ B. Adenocarcinoma

/ C. Barrett esophagus

/ D. Squamous cell carcinoma

/ E. Systemic sclerosis

Q 2

This patient's condition is most likely due to which of the following?

/ A. Acid reflux

/ B. Cancerous destruction

/ C. Candida infection

/ D. Fibrosis of the esophageal wall

/ E. Lack of ganglion cells

Q 3

Page 33: Abdominal Pain

Which of the following regulators would most likely inhibit the lower esophageal

sphincter in normal individuals?

/ A. Acetylcholine and substance P

/ B. Substance P and nitric oxide

/ C. Substance P only

/ D. Vasoactive intestinal polypeptide and acetylcholine

/ E. Vasoactive intestinal polypeptide and nitric oxide

Q4

Which of the following medications is used to directly relax the lower

esophageal sphincter?

/ A. Diphenoxylate

/ B. Famotidine

/ C. Granisetron

/ D. Isosorbide dinitrate

/ E. Metoclopramide

Q 5

Worldwide, which of the following parasitic diseases is most likely to produce a

disorder that clinically resembles this patient's condition?

/ A. Ascariasis

/ B. African sleeping sickness

/ C. Chagas disease

/ D. Cysticercosis

/ E. Malaria

Dysphagia case 3 answers

A1

The correct answer is A. The most likely diagnosis is achalasia. This condition is

a neurogenic esophageal disorder that can occur at any age, but frequently is

diagnosed when individuals are between the ages of 20 and 40.

Characteristically, the swallowing difficulties involve both solid food and liquids.

The manometry findings illustrated are typical; the barium swallow findings may

Page 34: Abdominal Pain

be as illustrated or may instead show diffuse esophageal dilation without the

"bird's beak" near the lower esophageal sphincter.

Larger cancers of the esophagus (choices B and D) would be more likely to

cause either a mass or an ulceration, which would be visible on barium swallow.

Very small cancers and Barrett's esophagus (choice C) would require

esophagogastroduodenoscopy with biopsy for diagnosis, and would be unlikely

to cause dysphagia.

Systemic sclerosis (choice E) can involve the esophagus as well, however,

physical examination would usually show obvious skin involvement.

A2

The correct answer is E. Individuals who have achalasia have been found to

have a deficiency of inhibitory ganglion cells within the esophageal wall. This

lack causes an imbalance in excitatory and inhibitory neurotransmission, with

the result that the lower esophageal sphincter tends to have a higher-than-

normal muscle tone and relaxes only with difficulty.

Acid reflux (choice A) can cause esophageal irritation, ulceration, and also

predisposes for Barrett's metaplasia with subsequent risk of adenocarcinoma of

the esophagus.

Cancerous destruction (choice B) would produce a mass, or area of stricture or

ulceration.

Candida infection (choice C) of the esophagus resembles thrush of the mouth,

and causes a usually superficial infection.

Fibrosis of the esophageal wall (choice D) can be the consequence of ulceration

(due to reflux or ingestion of harsh chemicals such as lye) or systemic sclerosis.

A3

The correct answer is E. Physiologically important inhibitors of the lower

esophageal sphincter include nitric oxide and vasoactive intestinal polypeptide.

Physiologically important substances that stimulate the lower esophageal

sphincter include acetylcholine and substance P.

A4

The correct answer is D. Commonly used medications to relax the lower

esophageal sphincter in patients with achalasia include nitrates such as

isosorbide dinitrate (remember that nitric oxide physiologically inhibits the lower

Page 35: Abdominal Pain

esophageal sphincter) and calcium channel blockers such as nifedipine (which

inhibit calcium flow into the smooth muscle of the lower esophageal sphincter,

thereby inhibiting contraction.) For patients in whom medical therapy fails, other

options include paralysis of the lower esophageal sphincter with intrasphincteric

injection of botulinum toxin, pneumatic dilatation, and a Heller myotomy (which

interrupts the muscles of the lower esophageal sphincter).

Diphenoxylate (choice A) is a opiate antidiarrheal, and would not be useful for

achalasia.

Famotidine (choice B) is an H2 antagonist that would be useful in reducing

stomach acidity, but would not directly affect lower esophageal pressure. Other

drugs in this class include cimetidine, ranitidine, and nizatidine.

Granisetron (choice C) is a 5HT3 antagonist and is used to prevent nausea and

vomiting in patients who receive chemotherapy, and after general anesthesia.

Other members of this drug class include ondansetron and dolasetron.

Metoclopramide (choice E) stimulates gastric motility in patients with

gastroparesis and is also a antiemetic agent.

A5

The correct answer is C. Chagas disease, which is found in South and Central

America and is due to infection with Trypanosoma cruzi, can involve the heart,

colon, and esophagus. The esophageal involvement clinically closely resembles

achalasia.

The adult worms of ascariasis (choice A), or roundworm infection, live principally

in the intestine, and can obstruct the intestine or a bile duct; the larvae can

migrate to the liver, heart, and lungs.

African sleeping sickness (choice B), caused by Trypanosoma brucei and

Trypanosoma gambiense, causes lymphadenopathy, rash, and CNS

involvement.

Cysticercosis (choice D), due to the larval form of the pork tapeworm Taenia

solium, can involve subcutaneous tissue, muscle, viscera (but not specifically

the esophagus), and, most seriously, the CNS.

Malaria (choice E), due to various Plasmodium species, involves the blood, liver,

kidney, spleen, and brain, but does not have a specific predilection for the

esophagus.

Page 36: Abdominal Pain

A 30-year-old man consults a physician because he has been having increasing difficulty swallowing both solids and liquids. Physical examination of the patient is noncontributory. Barium swallow studies show a mostly dilated esophagus with slow passage of barium into thestomach. The very distal part of the esophagus appears narrowed into a "bird's beak." Esophageal manometry shows incomplete relaxation of the lower esophageal sphincter in response to swallowing, high resting lower esophageal pressure, and absent esophageal peristalsis.

Q 1

The manometry and barium swallow studies most strongly support which of the following diagnoses?/ A. Achalasia/ B. Adenocarcinoma/ C. Barrett esophagus/ D. Squamous cell carcinoma/ E. Systemic sclerosis

Q 2This patient's condition is most likely due to which of the following?/ A. Acid reflux/ B. Cancerous destruction/ C. Candida infection/ D. Fibrosis of the esophageal wall/ E. Lack of ganglion cells

Q 3Which of the following regulators would most likely inhibit the lower esophageal sphincter in normal individuals?/ A. Acetylcholine and substance P/ B. Substance P and nitric oxide/ C. Substance P only/ D. Vasoactive intestinal polypeptide and acetylcholine/ E. Vasoactive intestinal polypeptide and nitric oxide

Q4Which of the following medications is used to directly relax the lower esophageal sphincter?/ A. Diphenoxylate

Page 37: Abdominal Pain

/ B. Famotidine/ C. Granisetron/ D. Isosorbide dinitrate/ E. Metoclopramide

Q 5Worldwide, which of the following parasitic diseases is most likely to produce a disorder that clinically resembles this patient's condition?/ A. Ascariasis/ B. African sleeping sickness/ C. Chagas disease/ D. Cysticercosis/ E. Malaria

Dysphagia case 3 answers

A1The correct answer is A. The most likely diagnosis is achalasia. This condition is a neurogenic esophageal disorder that can occur at any age, but frequently is diagnosed when individuals are between the ages of 20 and 40. Characteristically, the swallowing difficulties involve both solid food and liquids. The manometry findings illustrated are typical; the barium swallow findings may be as illustrated or may instead show diffuse esophageal dilation without the "bird's beak" near the lower esophageal sphincter.Larger cancers of the esophagus (choices B and D) would be more likely to cause either a mass or an ulceration, which would be visible on barium swallow.Very small cancers and Barrett's esophagus (choice C) would require esophagogastroduodenoscopy with biopsy for diagnosis, and would be unlikely to cause dysphagia.Systemic sclerosis (choice E) can involve the esophagus as well, however, physical examination would usually show obvious skin involvement.

A2The correct answer is E. Individuals who have achalasia have been found to have a deficiency of inhibitory ganglion cells within the esophageal wall. This lack causes an imbalance in excitatory and inhibitory neurotransmission, with

Page 38: Abdominal Pain

the result that the lower esophageal sphincter tends to have a higher-than-normal muscle tone and relaxes only with difficulty.Acid reflux (choice A) can cause esophageal irritation, ulceration, and also predisposes for Barrett's metaplasia with subsequent risk of adenocarcinoma of the esophagus.Cancerous destruction (choice B) would produce a mass, or area of stricture or ulceration.Candida infection (choice C) of the esophagus resembles thrush of the mouth, and causes a usually superficial infection.Fibrosis of the esophageal wall (choice D) can be the consequence of ulceration (due to reflux or ingestion of harsh chemicals such as lye) or systemic sclerosis.

A3The correct answer is E. Physiologically important inhibitors of the lower esophageal sphincter include nitric oxide and vasoactive intestinal polypeptide. Physiologically important substances that stimulate the lower esophageal sphincter include acetylcholine and substance P.

A4The correct answer is D. Commonly used medications to relax the lower esophageal sphincter in patients with achalasia include nitrates such as isosorbide dinitrate (remember that nitric oxide physiologically inhibits the lower esophageal sphincter) and calcium channel blockers such as nifedipine (which inhibit calcium flow into the smooth muscle of the lower esophageal sphincter, thereby inhibiting contraction.) For patients in whom medical therapy fails, other options include paralysis of the lower esophageal sphincter with intrasphincteric injection of botulinum toxin, pneumatic dilatation, and a Heller myotomy (which interrupts the muscles of the lower esophageal sphincter).Diphenoxylate (choice A) is a opiate antidiarrheal, and would not be useful for achalasia.Famotidine (choice B) is an H2 antagonist that would be useful in reducing stomach acidity, but would not directly affect lower esophageal pressure. Other drugs in this class include cimetidine, ranitidine, and nizatidine.Granisetron (choice C) is a 5HT3 antagonist and is used to prevent nausea and vomiting in patients who receive chemotherapy, and after general anesthesia. Other members of this drug class include ondansetron and dolasetron.Metoclopramide (choice E) stimulates gastric motility in patients with gastroparesis and is also a antiemetic agent.

A5

Page 39: Abdominal Pain

The correct answer is C. Chagas disease, which is found in South and Central America and is due to infection with Trypanosoma cruzi, can involve the heart, colon, and esophagus. The esophageal involvement clinically closely resembles achalasia.The adult worms of ascariasis (choice A), or roundworm infection, live principally in the intestine, and can obstruct the intestine or a bile duct; the larvae can migrate to the liver, heart, and lungs.African sleeping sickness (choice B), caused by Trypanosoma brucei and Trypanosoma gambiense, causes lymphadenopathy, rash, and CNS involvement.Cysticercosis (choice D), due to the larval form of the pork tapeworm Taenia solium, can involve subcutaneous tissue, muscle, viscera (but not specifically the esophagus), and, most seriously, the CNS.Malaria (choice E), due to various Plasmodium species, involves the blood, liver, kidney, spleen, and brain, but does not have a specific predilection for the esophagus.

TOTAL 43 Questions Abdominal Pain, Actomenaphin, Achlasia, Acid reflux

A 53-year-old man consults a physician because he has begun coughing up

sputum tinged with fresh blood. He does not initially report any other symptoms

to his physician. When his physician comments on his hoarseness and cough,

the patient discounts these symptoms and attributes them to his long smoking

history. He cannot say when they began or became worse.

Q 1

The chronic hoarseness suggests dysfunction of which of the following?

/ A. Palate

/ B. Pharynx

/ C. Tongue

/ D. Trachea

/ E. Vocal cords

Q 2

Laryngoscopy reveal a fungating tumor of the larynx that is located between the

false and true vocal cords. This tumor is in which of the following sites?

/ A. Aryepiglottic fold

/ B. Infraglottic compartment

/ C. Piriform recess

Page 40: Abdominal Pain

/ D. Supraglottic compartment

/ E. Ventricle

Q 3

Biopsy of the mass demonstrates a malignancy. Which of the following is the

most likely diagnosis?

/ A. Adenocarcinoma

/ B. Lymphoma

/ C. Oat cell carcinoma

/ D. Sarcoma

/ E. Squamous cell carcinoma

Q 4

In addition to smoking, which of the following is an accepted risk factor for this

patient's tumor?

/ A. Alcohol use

/ B. Cocaine use

/ C. Coffee use

/ D. Marijuana use

/ E. Tea use

Q 5

The patient returns to clinic three weeks after receiving the news that his cancer

is inoperable. His wife reports that he has been more withdrawn, eating and

sleeping poorly and "just seems to have lost all hope." On examination, the

patient moves very little, never makes eye contact, and admits to "some"

depressed mood, Ioss of appetite, and sleep disruption. Which of the following

symptoms is of most concern regarding his risk of suicide?

/ A. Decreased appetite

/ B. Decreased energy

/ C. Diminished concentration

/ D. Guilty and worthless feelings

/ E. Hopelessness

Q 6

Some patients present with hoarseness due to a tumor at the lung apex that

Page 41: Abdominal Pain

involves a nerve that is a branch of which of the following?

/ A. Accessory nerve

/ B. Glossopharyngeal nerve

/ C. Hypoglossal nerve

/ D. Phrenic nerve

/ E. Vagus nerve

____________________________________________________________________

Hemoptysis Case 3 Answers

A1

The correct answer is E. Hoarseness specifically suggests dysfunction of the

vocal cords, which produce the sounds that are then articulated to speech with

the pharynx (choice B), palate (choice A), tongue (choice C), teeth, and lips.

While vocal cord dysfunction is specifically suggested by hoarseness, the

anatomic lesion may or may not be present at the level of the vocal cords. Vocal

paralysis may be the result of local tumor or trauma, intracranial lesions

affecting the nucleus ambiguus or its supranuclear tracts, and lesions at the

base of the skull, neck, or upper portion of the thorax that involve either the

vagus nerve or the recurrent laryngeal nerves.

Air from the trachea (choice D) passes into the larynx to reach the vocal cords,

but tracheal dysfunction does not produce hoarseness.

A2

The correct answer is E. The interior of the larynx is divided into 3

compartments: the supraglottic compartment (choice D) above the false vocal

cords, the ventricle between the false and true vocal cords, and the infraglottic

compartment (choice B) below the true vocal cords.

The aryepiglottic fold (choice A) is the upper free border of the quadrangular

membrane found in the supraglottic compartment.

The piriform recess (choice C) lies behind the thyroid laminae and the lateral

wall of the supraglottic compartment.

A3

The correct answer is E. The epithelial lining of the larynx is squamous

epithelium, and the vast majority of cancers of the larynx are squamous cell

Page 42: Abdominal Pain

carcinomas. Squamous cell carcinoma can present with hoarseness, cough,

hemoptysis, or difficulty swallowing. If the symptoms develop insidiously (as in

the case), the patient may not seek medical attention until late in the course.

While very small laryngeal cancers can be treated successfully with surgery

and/or radiation, larger ones are much more problematic, in large part because

complete resection of the cancer and any lymph node metastases can be

difficult to impossible to perform without compromising the many vital neck

structures. Patients treated with partial laryngectomy may retain some speech

ability. Patients treated with total laryngectomy can often learn to speak again

using esophageal speech (gradual belching of air through the

pharyngoesophageal junction), a tracheoesophageal fistula (one way valve

between the trachea and the esophagus which makes a sound when air is

forced across it), or an electrolarynx (sound source held against the neck). With

all three techniques, the sound produced is then turned into articulation by the

patient's pharynx, palate, tongue, teeth, and lips. The other tumors listed in the

choices are very uncommon in the larynx.

A4

The correct answer is A. The only two risk factors that you will need to associate

with laryngeal carcinoma are smoking and alcohol use. Cocaine (choice B),

coffee (choice C), and tea (choice E) have not been linked to laryngeal cancer.

Marijuana (choice D) has been suggested as a risk factor for oral cancer in a few

cases, but a causal association has not been established.

A5

The correct answer is E. Hopelessness is an ominous sign and is associated with

a higher risk for suicide.

Decreased appetite (choice A), decreased energy (choice B), and diminished

concentration (choice C) are symptoms indicating the presence of a major

depressive episode, or are the result of the medical illness.

Guilty and worthless feelings (choice D) are symptoms of depression. They do

not predict risk for suicide.

A6

The correct answer is E. The larynx has complex innervation. The vocal cords

and most of the area of the larynx below them are supplied by the recurrent

Page 43: Abdominal Pain

laryngeal nerve, which is a branch of the vagus nerve that passes beneath the

subclavian artery before returning to the neck to innervate the larynx. From

above the larynx, the superior laryngeal nerve arises from the vagus and divides

into the internal laryngeal nerve and the external laryngeal nerve. The

innervation above the vocal cords is by the internal laryngeal branch of the

vagus nerve. The cricothyroid muscle (the only muscle of the larynx not supplied

by the recurrent laryngeal nerve) is supplied by the external laryngeal branch of

the superior laryngeal, which also branches off the vagus, but which contains

motor fibers originally derived from the accessory nerve (choice A).

The glossopharyngeal nerve (choice B) supplies the pharynx.

The hypoglossal nerve (choice C) supplies the tongue.

The phrenic nerve (choice D) supplies the diaphragm.

Posted by admin at 8:26 AM 0 comments

Labels: adenocarcinoma, Alcohol, Cocaine, Hemoptysis, Lymphoma, malignancy,

Marijuana, Sarcoma, Squamous cell carcinoma

Hemoptysis Case 2

A fifty-year-old man presents to his primary care doctor complaining of chronic

cough with hemoptysis, weakness, frequent urination, thirst, and a decreased

ability to concentrate. Review of systems reveals fever, chills, night sweats, and

a twenty-pound weight loss. He has a

seventy pack-year smoking history. On examination, his lungs are clear to

auscultation, but neurologic examination reveals global hyporeflexia.

A chest x-ray film reveals a 4-cm hilar non-cavitary opacity in the left lung.

Serum electrolytes show K+ = 2.3 mEq/L. Calcium is within normal limits.

Q 1

Which of the following is the most likely diagnosis?

/ A. Adenocarcinoma

/ B. Bronchioloalveolar carcinoma

/ C. Mesothelioma

/ D. Small cell carcinoma

Page 44: Abdominal Pain

/ E. Squamous cell carcinoma

Q 2

Which of the following substances secreted from the lesion is the most likely

cause of this patient's electrolyte disturbance?

/ A. ACTH

/ B. Aldosterone

/ C. Angiotensin ll

/ D. Epinephrine

/ E. PTH

Q 3

As the patient's condition progresses, the patient grows increasingly hoarse.

Which of the following is the most likely reason for the patient's hoarseness?

/ A. Enlargement of the mass has severely decreased inspiratory volume.

/ B. Extension of the mass into the larynx

/ C. Impingement on cranial nerve XII

/ D. Impingement on the recurrent laryngeal nerve

/ E. Metastasis into the speech centers of the brain

Q 4

This patient is given cisplatin as part of his chemotherapeutic regimen. Which of

the following is a known adverse effect of cisplatin?

/ A. Cardiotoxicity

/ B. Hemorrhagic cystitis

/ C. Nephrotoxicity

/ D. Profound myelosuppression

/ E. Pulmonary fibrosis

Q 5

In addition, this patient is given etoposide. Which of the following best describes

the mechanism of action of etoposide?

/ A. Etoposide cross-Iinks DNA

/ B. Etoposide induces single- and double-stranded breaks in DNA

/ C. Etoposide inhibits dihydrofolate reductase

/ D. Etoposide inhibits microtubule formation

Page 45: Abdominal Pain

/ E. Etoposide inhibits topoisomerase ll

Q 6

The patient has been aware of his diagnosis and prognosis for several weeks

now. He makes the statement "I had successes and failures but I'm pretty sure l

got as much living out of the last 50 years as anybody could." This statement

suggests that the patient is experiencing which of the following of Erikson's

stages?

/ A. Ego integrity vs. despair

/ B. Generativity vs. self absorption

/ C. Identity vs. role confusion

/ D. Industry vs. inferiority

/ E. Intimacy vs. isolation

____________________________________________________________________

Hemoptysis Case 2 Answers

A1

The correct answer is D. This patient has classic findings for small cell carcinoma

of the lung (also known as oat cell carcinoma). Patients with small cell

carcinoma typically present with constitutional symptoms, cough, and

hemoptysis. Paraneoplastic syndromes are common with this cancer, and may

often be the first symptoms to present. This patient has hypokalemia and many

of the symptoms that accompany it: muscular weakness, frequent urination,

thirst, and decreased ability to concentrate. This patient's radiographic findings

(non-cavitary hilar mass) also support the diagnosis of small cell carcinoma.

Adenocarcinomas (choice A) tend to present as peripheral masses, and

paraneoplastic syndromes are rare.

Bronchioloalveolar carcinoma (choice B) is a subtype of adenocarcinoma. It

arises from the peripheral airways, and while it can present as a discrete mass,

it typically is indistinct radiographically as it grows down the airway surfaces.

Again, paraneoplastic syndromes are rare.

Mesothelioma (choice C) is a pleural malignancy associated with asbestos

exposure. They do not present as hilar masses. 50% metastasize, but death

usually results from local extension. They tend to produce large effusions that

may obscure the mass on chest x-ray films.

Squamous cell carcinoma (choice E), like small cell carcinoma, has a strong

Page 46: Abdominal Pain

association with smoking. These tumors may present in a similar fashion, with a

more central lung lesion, but paraneoplastic syndromes with squamous cell

carcinoma tend to be related to the secretion of a PTH-like substance, resulting

in hypercalcemia. Thus the diagnosis of small cell carcinoma is more likely.

A2

The correct answer is A. ACTH is often secreted by small cell lung carcinoma.

The clinical results are those seen from uninhibited ACTH secretion: the adrenal

gland increases the production of glucocorticoids and mineralocorticoids. Thus,

patients experience hypokalemia, hyperglycemia, and hypertension. The chronic

sequelae of this process (Cushingoid symptoms) are not usually seen, due to the

metabolic demands of the neoplasm and the short survival of affected

individuals.

None of the other hormones listed above are commonly secreted by small cell

cancers.

A3

The correct answer is D. As seen with many hilar masses, impingement on the

recurrent laryngeal nerve produces hoarseness. The recurrent laryngeal nerve

courses inferiorly to the aortic arch and then turns superiorly to innervate the

intrinsic muscles of the larynx (except the cricothyroid muscle). When this nerve

sustains unilateral damage, hoarseness results, as patients have difficulty

abducting the vocal cords. Bilateral damage results in acute breathlessness,

because both of the vocal cords move to the midline and block the airway.

Enlargement of the mass (choice A) to a size that would affect speech would

likely produce pulmonary collapse, not hoarseness.

Extension of the mass from the hilum of the lung to the larynx (choice B) would

be unlikely. Many critical structures are in this course, and the patient would

likely not survive long enough to become hoarse from direct extension.

Impingement on cranial nerve XII (choice C) would affect movement of the

tongue. Not only would metastasis or direct extension to the anatomic site of

this nerve be unlikely, this event would not make a patient hoarse.

Metastasis to the speech centers (choice E) would produce more profound

speech difficulties. The characteristics of the aphasia would depend upon which

speech center is affected ( i.e., fluent vs. expressive aphasia).

Page 47: Abdominal Pain

A4

The correct answer is C. Cisplatin is a heavy metal compound used in the

treatment of small cell carcinoma of the lung. Cisplatin cross-links DNA, though

it is not a true alkylating agent. The dominant adverse effect seen with its

administration is nephrotoxicity, as it is toxic to both proximal and distal renal

tubule epithelium. It produces only modest myelosuppression.

Doxorubicin causes cardiotoxicity (choice A) and congestive heart failure may

result.

Cyclophosphamide is a chemotherapeutic agent known to cause hemorrhagic

cystitis (choice B).

The myelosuppression with cisplatin is modest. Many other chemotherapeutic

agents are known to severely suppress the marrow (choice D).

Bleomycin is known to cause pulmonary fibrosis (choice E).

A5

The correct answer is E. Etoposide acts by inhibiting the enzyme topoisomerase

II. Topoisomerases are enzymes that create and repair breaks in DNA during

replication. This enzyme is needed to relieve topologic and conformational

changes as the DNA is "unzipped" during replication and transcription.

Alkylating agents, such as cyclophosphamide, cross-link DNA (choice A).

The antitumor antibiotics, such as bleomycin, act by inducing breaks in DNA

(choice B).

Methotrexate is an antimetabolite chemotherapeutic agent that inhibits

dihydrofolate reductase (choice C), an enzyme needed to produce purine

nucleotides. Thus the "purine shortage" produced hinders cell replication.

The Vinca alkaloids exert their effects by inhibiting microtubule formation

(choice D). Without microtubules, cells cannot mobilize their chromosomes, and

thus mitosis is inhibited.

A6

The correct answer is A. In the ego integrity vs. despair stage, individuals

reconcile their achievements and failures and face the fact that any human life

is limited. Persons completing this task find self-worth in reviewing their life

events. Persons stuck at this task cannot admit that time has run out.

Generativity vs. self-absorption (choice B) usually occurs from age 30-65. It

involves noting ones contributions as a parent (in the broadest sense) and

Page 48: Abdominal Pain

eventually "passing the torch" to the next generation. Persons stuck in this

phase refuse to give up any power to their successors, and maintain ultimate

authority without making a place for the next generation.

Identity vs. role confusion (choice C) occurs during adolescence, and is usually

defined by emotional and or geographic separation from the actual parents, with

an internal identity based on both similarities and differences from parental

traits and values.

Industry vs. inferiority (choice D) is a stage in which school age (6-12) children

explore peer groups and learn that industry (hard work) generally pays off.

Intimacy vs. isolation (choice E) is the stage, in the early twenties, when young

adults connect to their peers and significant others in a meaningful and mature

way, putting aside fear and inhibitions.

Posted by admin at 8:14 AM 0 comments

Labels: adenocarcinoma, Asbestos, Bronchioloalveolar carcinoma, Epinephrine,

Hemoptysis, hoarseness, malignancy, Mesothelioma, Pulmonary fibrosis, Small

cell carcinoma, Squamous cell carcinoma

S A T U R D A Y , A P R I L 1 0 , 2 0 1 0

Dysphagia case 4

A 35-year-old woman consults a physician because she has been having trouble

swallowing. She also often experiences chronic heartburn. The physician

performs a screening physical examination, and notices that the skin of her

hands appears tight and shiny. On specific

questioning, she reports having often experienced color changes in her hands

from white to blue to red.

Q1

Which of the following is the most likely cause of the patient's difficulties with

swallowing?

/ A. Achalasia

/ B. Adenocarcinoma

Page 49: Abdominal Pain

/ C. Chagas disease

/ D. Scleroderma

/ E. Squamous carcinoma

Q2

The color changes described on the patient's hand are most likely due to which

of the following?

/ A. Arteriolar spasm

/ B. Blood clots at sites of vascular injury

/ C. Large artery spasm

/ D. Platelet clots

/ E. Stasis blood clots

Q 3

Additional findings on physical examination include noting that the skin changes

are limited to areas distal to the elbow and knee, the presence of calcified

nodules on the extensor surfaces of the forearms, and the presence of

telangiectasias on the forearms. This suggests

that this patient has which of the following?

/ A. Bauer syndrome

/ B. Charcot syndrome

/ C. CREST syndrome

/ D. Crigler-Najjar syndrome

/ E. Dandy-Walker syndrome

Q 4

If this woman's involved skin were biopsied, which of the following would most

likely be seen?

/ A. Cleft separating the dermis and subcutaneous tissues

/ B. Epithelial cell hyperplasia

/ C. Marked dermal fibrosis

/ D. Narrowing of the basal lamina of small capillaries

/ E. Thickening of rete pegs

Page 50: Abdominal Pain

Q 5

More than 90% of the patients with the limited cutaneous form of this disorder

make which of the following autoantibodies?

/ A. Anti-centromere

/ B. Anti-DNA topoisomerase l

/ C. Anti-double-stranded DNA

/ D. Anti-Golgi

/ E. Anti-ScI-70

Dysphagia case 4 answers

A1

The correct answer is D. The tip-off is the reference to the patient's skin changes

that are typical for scleroderma, also known as systemic sclerosis. Scleroderma

is a disease that may be either predominately limited to the skin or involve

many body systems, including the musculoskeletal system, gastrointestinal tract

(with esophageal involvement most often symptomatic), cardiorespiratory

system, and renal system. Esophageal dysfunction is a common complication of

scleroderma. In most patients, the skin changes are obvious, even if the patient

has not been previously diagnosed. Rarely, the skin changes may be noticed at

an earlier stage, in which the skin of the hands appears puffy and edematous,

but not scarred.

Achalasia (choice A) and Chagas disease (choice C) are also important causes of

dysphagia, but in achalasia, the physical examination is usually normal, and in

Chagas disease, you should be able to elicit a history of possible exposure in

Central or South America.

Reflux esophagitis with risk of progression to Barrett esophagus and

adenocarcinoma (choice B) can occur in scleroderma, but it is unlikely, at this

early stage in the patient's disease, that she already has adenocarcinoma.

The risk of squamous cell carcinoma (choice E) is not increased in this patient.

A2

The correct answer is A. The color changes described are typical for Raynaud's

phenomenon, which occurs because of changes in perfusion due to arteriolar

spasm. Raynaud's phenomenon is common in scleroderma, largely because the

subintimal hyperplasia of small vessels characteristic of scleroderma can reduce

Page 51: Abdominal Pain

the luminal diameter by more than 75%. Some authors argue that the vascular

changes seen in scleroderma are actually the insult that triggers the subsequent

development of fibrosis.

Blood clots at sites of vascular injury (choice B) are typical in clots that form in

arteries on surfaces such as a fractured atherosclerotic clot.

Larger artery spasm (choice C) of vessels damaged by atherosclerosis is thought

to contribute to some myocardial infarctions, but is not thought to be an

important pathologic mechanism in scleroderma.

Platelet clots (choice D) can be seen in some diseases in which platelet function

is abnormal, such as essential thrombocytopenia.

Stasis blood clots (choice E) commonly are found in venous thromboses.

A3

The correct answer is C. These findings, together with esophageal dysfunction

and Raynaud's phenomenon (both of which this patient has), are called the

CREST syndrome, also known as limited cutaneous scleroderma. This form of

scleroderma has a better long-term prognosis than when the skin changes also

involve the trunk (diffuse scleroderma) and more internal organs are

additionally involved.

Bauer syndrome (choice A) is aortitis and aortic endocarditis as a complication

of rheumatoid arthritis.

Charcot syndrome (choice B) is intermittent claudication.

Crigler-Najjar syndrome (choice D) is a severe familial liver disease.

Dandy-Walker syndrome (choice E) is a malformation of the central nervous

syndrome.

A4

The correct answer is C. In scleroderma, early changes (at the point at which the

hands appear swollen, rather than with tight, thick skin) show edema with

perivascular infiltrates of CD4+ T cells. At this stage, the collagen fibers are

swollen and beginning to degenerate. The smaller vessels may show basal

lamina thickening (not narrowing as in choice D) and endothelial (not epithelial

Page 52: Abdominal Pain

cell hyperplasia) cell damage and proliferation. With time, the characteristic

marked dermal fibrosis develops, which tends to both narrow (not thicken as in

choice E) the rete pegs and attach the dermis tightly (compare with cleft

formation as in choice A) to subcutaneous tissues.

A5

The correct answer is A. All forms of scleroderma are thought to have a strong

autoimmune component, and glucocorticoids and azathioprine are used to

suppress the inflammatory complications of scleroderma. (Other drugs that can

be used in therapy include penicillamine, which inhibits collagen cross-linking,

NSAIDS for pain, and ACE inhibitors to protect the kidney if hypertension or renal

damage occurs.) The anti-centromere antibody is quite specific for CREST

syndrome (96% of cases), and is only seen in a minority of patients with diffuse

scleroderma (mainly those with Raynaud's phenomenon) and rarely in systemic

lupus erythematosus and mixed connective tissue disease.

Anti-DNA topoisomerase I (choice B), also called anti-Scl-70 (choice E) occurs

commonly (64-75%) in diffuse scleroderma, but only rarely in CREST syndrome.

Anti-double-stranded DNA (choice C) is fairly specific for systemic lupus

erythematosus, although it only occurs in 50-60% of lupus cases.

Anti-Golgi antibodies (choice D) are seen most often in systemic lupus

erythematosus and Sjögren syndrome.

Dysphagia case 3

A 30-year-old man consults a physician because he has been having increasing

difficulty swallowing both solids and liquids. Physical examination of the patient

is noncontributory. Barium swallow studies show a mostly dilated esophagus

Page 53: Abdominal Pain

with slow passage of barium into the

stomach. The very distal part of the esophagus appears narrowed into a "bird's

beak." Esophageal manometry shows incomplete relaxation of the lower

esophageal sphincter in response to swallowing, high resting lower esophageal

pressure, and absent esophageal peristalsis.

Q 1

The manometry and barium swallow studies most strongly support which of the

following diagnoses?

/ A. Achalasia

/ B. Adenocarcinoma

/ C. Barrett esophagus

/ D. Squamous cell carcinoma

/ E. Systemic sclerosis

Q 2

This patient's condition is most likely due to which of the following?

/ A. Acid reflux

/ B. Cancerous destruction

/ C. Candida infection

/ D. Fibrosis of the esophageal wall

/ E. Lack of ganglion cells

Q 3

Which of the following regulators would most likely inhibit the lower esophageal

sphincter in normal individuals?

/ A. Acetylcholine and substance P

/ B. Substance P and nitric oxide

/ C. Substance P only

/ D. Vasoactive intestinal polypeptide and acetylcholine

/ E. Vasoactive intestinal polypeptide and nitric oxide

Q4

Which of the following medications is used to directly relax the lower

esophageal sphincter?

/ A. Diphenoxylate

/ B. Famotidine

Page 54: Abdominal Pain

/ C. Granisetron

/ D. Isosorbide dinitrate

/ E. Metoclopramide

Q 5

Worldwide, which of the following parasitic diseases is most likely to produce a

disorder that clinically resembles this patient's condition?

/ A. Ascariasis

/ B. African sleeping sickness

/ C. Chagas disease

/ D. Cysticercosis

/ E. Malaria

Dysphagia case 3 answers

A1

The correct answer is A. The most likely diagnosis is achalasia. This condition is

a neurogenic esophageal disorder that can occur at any age, but frequently is

diagnosed when individuals are between the ages of 20 and 40.

Characteristically, the swallowing difficulties involve both solid food and liquids.

The manometry findings illustrated are typical; the barium swallow findings may

be as illustrated or may instead show diffuse esophageal dilation without the

"bird's beak" near the lower esophageal sphincter.

Larger cancers of the esophagus (choices B and D) would be more likely to

cause either a mass or an ulceration, which would be visible on barium swallow.

Very small cancers and Barrett's esophagus (choice C) would require

esophagogastroduodenoscopy with biopsy for diagnosis, and would be unlikely

to cause dysphagia.

Systemic sclerosis (choice E) can involve the esophagus as well, however,

physical examination would usually show obvious skin involvement.

A2

Page 55: Abdominal Pain

The correct answer is E. Individuals who have achalasia have been found to

have a deficiency of inhibitory ganglion cells within the esophageal wall. This

lack causes an imbalance in excitatory and inhibitory neurotransmission, with

the result that the lower esophageal sphincter tends to have a higher-than-

normal muscle tone and relaxes only with difficulty.

Acid reflux (choice A) can cause esophageal irritation, ulceration, and also

predisposes for Barrett's metaplasia with subsequent risk of adenocarcinoma of

the esophagus.

Cancerous destruction (choice B) would produce a mass, or area of stricture or

ulceration.

Candida infection (choice C) of the esophagus resembles thrush of the mouth,

and causes a usually superficial infection.

Fibrosis of the esophageal wall (choice D) can be the consequence of ulceration

(due to reflux or ingestion of harsh chemicals such as lye) or systemic sclerosis.

A3

The correct answer is E. Physiologically important inhibitors of the lower

esophageal sphincter include nitric oxide and vasoactive intestinal polypeptide.

Physiologically important substances that stimulate the lower esophageal

sphincter include acetylcholine and substance P.

A4

The correct answer is D. Commonly used medications to relax the lower

esophageal sphincter in patients with achalasia include nitrates such as

isosorbide dinitrate (remember that nitric oxide physiologically inhibits the lower

esophageal sphincter) and calcium channel blockers such as nifedipine (which

inhibit calcium flow into the smooth muscle of the lower esophageal sphincter,

thereby inhibiting contraction.) For patients in whom medical therapy fails, other

options include paralysis of the lower esophageal sphincter with intrasphincteric

injection of botulinum toxin, pneumatic dilatation, and a Heller myotomy (which

interrupts the muscles of the lower esophageal sphincter).

Diphenoxylate (choice A) is a opiate antidiarrheal, and would not be useful for

achalasia.

Famotidine (choice B) is an H2 antagonist that would be useful in reducing

Page 56: Abdominal Pain

stomach acidity, but would not directly affect lower esophageal pressure. Other

drugs in this class include cimetidine, ranitidine, and nizatidine.

Granisetron (choice C) is a 5HT3 antagonist and is used to prevent nausea and

vomiting in patients who receive chemotherapy, and after general anesthesia.

Other members of this drug class include ondansetron and dolasetron.

Metoclopramide (choice E) stimulates gastric motility in patients with

gastroparesis and is also a antiemetic agent.

A5

The correct answer is C. Chagas disease, which is found in South and Central

America and is due to infection with Trypanosoma cruzi, can involve the heart,

colon, and esophagus. The esophageal involvement clinically closely resembles

achalasia.

The adult worms of ascariasis (choice A), or roundworm infection, live principally

in the intestine, and can obstruct the intestine or a bile duct; the larvae can

migrate to the liver, heart, and lungs.

African sleeping sickness (choice B), caused by Trypanosoma brucei and

Trypanosoma gambiense, causes lymphadenopathy, rash, and CNS

involvement.

Cysticercosis (choice D), due to the larval form of the pork tapeworm Taenia

solium, can involve subcutaneous tissue, muscle, viscera (but not specifically

the esophagus), and, most seriously, the CNS.

Malaria (choice E), due to various Plasmodium species, involves the blood, liver,

kidney, spleen, and brain, but does not have a specific predilection for the

esophagus.

Dysphagia case 2

A 65-year-old man presents to his physician because he has been having

increasing difficulty swallowing over the past 2 months. He is still able to

swallow liquids, but swallowing solid food now causes severe pain and a sense

of fullness behind his sternum. He has lost 18 pounds

since his swallowing difficulties began. The patient is referred to a

gastroenterologist, who demonstrates a mass lesion of the distal esophagus,

which on biopsy is shown to contain cancer.

Page 57: Abdominal Pain

Q1

Which of the following is most important in separating the esophagus from the

larynx, and must consequently be carefully passed behind

during endoscopy?

/ A. Arytenoids

/ B. Cricoid cartilage

/ C. Epiglottis

/ D. Pharynx

/ E. Vocal cords

Q2

Which of the following nerves provides the efferent impulses necessary for the

esophageal actions that occur during swallowing?

/ A. Glossopharyngeal

/ B. Hypoglossal

/ C. Spinal accessory

/ D. Trigeminal

/ E. Vagus

Q3

Which of the following approximately represents the proportion of different

esophageal cancer types now being observed in the United States?

/ A. 1/10 adenocarcinoma and 9/10 squamous cell carcinoma

/ B. 1/3 adenocarcinoma and 2/3 squamous cell carcinoma

/ C. 1/2 adenocarcinoma and 1/2 squamous cell carcinoma

/ D. 2/3 adenocarcinoma and 1/3 squamous cell carcinoma

/ E. 9/10 adenocarcinoma and 1/10 squamous cell carcinoma

Page 58: Abdominal Pain

Q 4

Precancerous metaplasia of the esophageal epithelium gives rise to a mucosa

resembling which of the following?

/ A. Mesothelium

/ B. Respiratory epithelium

/ C. Small intestine

/ D. Squamous epithelium

/ E. Stomach

Q5

Frequent use of which of the following has recently been found to probably have

a protective effect against development of esophageal

cancer?

/ A. Acetaminophen

/ B. Alcohol

/ C. Aspirin

/ D. Cigarettes

/ E. Codeine

Q 6

Currently, esophageal cancer has which of the following long-term survival

rates?

/ A. Less than 5%

/ B. 30%

/ C. 50%

/ D. 70%

/ E. More than 95%

Page 59: Abdominal Pain

Dysphagia case 2 answers

A1

The correct answer is C. Endoscopists are very careful when guiding the

endoscope past the epiglottis, which is a pear-shaped portion of elastic cartilage

that can be moved during swallowing to close the larynx, preventing swallowed

material from eventually entering the lungs.

The arytenoids (choice A) are the site of the attachment of the vocal cords

(choice E) within the larynx.

The cricoid cartilage (choice B) is in the more distal portion of the larynx.

The pharynx (choice D) is shared by the respiratory and gastrointestinal tracts.

A2

The correct answer is E. The vagus nerve supplies the efferent input into the

esophagus that is necessary for swallowing.

The glossopharyngeal nerve (choice A) provides taste and sensation on the

palate, but the only muscle it supplies is the stylopharyngeus.

The hypoglossal nerve (choice B) moves the tongue during the initiation of

swallowing, but does not innervate the esophagus.

The spinal accessory nerve (choice C) plays no role in swallowing. This nerve

mediates head and shoulder movement and innervates laryngeal muscles.

The trigeminal nerve (choice D) provides general sensation to the mouth and

motor innervation to the muscles of mastication.

A3

The correct answer is C. More recent statistics indicate that the incidence of

adenocarcinoma and squamous cell carcinoma of the esophagus are now

roughly equal. Formerly, approximately 2/3 of the esophageal cancers were

squamous in origin (choice B). Adenocarcinoma of the esophagus is often found

in the distal esophagus.

Page 60: Abdominal Pain

A4

The correct answer is C. This is an indirect question about Barrett's esophagus,

which is an important precursor of adenocarcinoma of the esophagus. While

Barrett's esophagus was initially defined to be either gastric-type or intestinal-

type metaplasia of the esophagus, more recent studies have shown that the

actual problem lesion is more likely to be intestinal metaplasia (diagnosed when

isolated goblet cells are seen in the epithelium) rather than gastric metaplasia

(choice E).

Metaplasia to mesothelium (choice A) or ciliated respiratory epithelium (choice

B) does not usually occur in the esophagus.

The normal epithelium of most of the esophagus is squamous (choice D).

A5

The correct answer is C. An interesting new research observation that may be

exploited in the future is that the incidence of esophageal cancer appears to be

much lower in people who use aspirin frequently.

Cigarettes (choice D) and alcohol use (choice B) have been implicated as risk

factors for esophageal cancer.

Acetaminophen (choice A) and codeine (choice E) have no known effects on the

incidence of esophageal cancer.

A6

The correct answer is A. Esophageal cancer is one of the very bad cancers,

presently with poor long-term survival. The underlying problem is that the

esophagus is only about 3 mm thick, and both metastatic disease and direct

spread (often unresectable) to mediastinal structures is common. Active

research is presently being undertaken to modify this prognosis by using

chemotherapy and radiation therapy prior to surgery, but these modalities have

not yet come into widespread use.

Page 61: Abdominal Pain

A 47-year-old, darkly pigmented man with a known history of alcohol abuse begins vomiting

large quantities of blood and is brought by ambulance to the emergency department.

Q 1

In the emergency department, the man is found to have a temperature of 36.7 C (98.1 F), blood

pressure of 65/40 mm Hg and dropping rapidly, a weak pulse of 130/min, and respirations of

29/min. These vital signs suggest that which of the following is developing?

/ A. Congestive heart failure

/ B. Meningitis

/ C. Pneumonia

/ D. Septicemia

/ E. Shock

Q 2

A blood sample is drawn and an IV Iine is started. While the patient is being cross-matched, the

physical examination is continued. The patient's sclerae are noted to be icteric and his nail beds

and palms have a yellowish hue. A caput medusa

is noted. Which of the following is the most accurate description of a caput medusa?

/ A. Ecchymoses over the mastoid process

/ B. Paradoxical increase in venous distension and pressure during inspiration

/ C. Reflex movement of the eyes in the opposite direction to that in which the head is moved

/ D. Small bony masses found on the terminal phalanges

/ E. Varicose veins radiating from the area of the umbilicus

Page 62: Abdominal Pain

Q 3

Caput medusa specifically suggests which of the following diagnoses?

/ A. BIadder infection

/ B. Duodenal ulcer

/ C. Gastric ulcer

/ D. Pancreatitis

/ E. Portal hypertension

Q 4

Which of the following is the most common cause of this patient's disorder in the United States?

/ A. Hepatic cirrhosis

/ B. Hepatic vein thrombosis

/ C. Hepatocellular carcinoma

/ D. Metastatic disease to the liver

/ E. Portal vein thrombosis

Q 5

Endoscopic studies demonstrate that this patient has bleeding esophageal varices, and the

bleeding is successfully stopped with

sclerotherapy. What percentage of patients with bleeding esophageal varices have another

episode of variceal bleeding at a subsequent

time?

/ A. 5%

Page 63: Abdominal Pain

/ B. 25%

/ C. 40%

/ D. 70%

/ E. 95%

Q 6

Following blood transfusions and sclerotherapy, the patient initially feels reasonably well and is

able to converse with medical personneI. Over

the next 12 hours, while he does not begin to rebleed, his mental status deteriorates. Arterial

blood levels of which of the following would be

most helpful in confirming the likely diagnosis?

/ A. Ammonia

/ B. Angiotensin l

/ C. Calcitonin

/ D. Carbon monoxide

/ E. Ceruloplasmin

Gastrointestinal Bleeding case 5 answers

A1

The correct answer is E. The patient's low and dropping blood pressure, tachycardia, high respiratory

rate, and slightly below normal body temperature are all consistent with impending shock. At this point,

the other conditions listed in the choices have not yet been ruled out, but clinically, the patient should

begin to be immediately treated for the shock, even if the therapeutic workup for underlying conditions

must be temporarily deferred

Page 64: Abdominal Pain

A2

The correct answer is E. Medusa was a goddess with snakes instead of hair

on her head. The caput medusa (Medusa's head) is an old term still in fairly

common use for numerous varicose veins radiating over the abdomen from the

area of the umbilicus.

Choice A describes Battle's sign, which is suggestive of basal skull fracture.

Choice B describes Kussmaul's sign, which is seen in constrictive pericarditis.

Choice C describes the doll's eye sign, which is looked for in the evaluation of

comatose patients and suggests functional integrity of the brainstem tegmental

pathways and cranial nerves involved in eye movement.

Choice D describes Heberden's nodules, which are seen in osteoarthritis.

A3

The correct answer is E. The caput medusa develops when severe portal hypertension induces dilation

of the anastomotic channels between the portal venous system and the systemic venous system, some

of which involve the superficial veins near the umbilicus. The other answers are distracters.

A4

The correct answer is A. The overwhelmingly most common cause of portal hypertension in the United

States is hepatic cirrhosis, which is usually due to either alcoholism or hepatitis viral infection. In this

patient's case, the diagnosis of cirrhosis is further clinically substantiated by his jaundice, as evidenced

by his sclera, nail beds, and palms. (Look in these areas on individuals in whom dark skin pigmentation

may mask the jaundice generally.) The other entities listed are occasional causes of portal hypertension.

A5

Page 65: Abdominal Pain

The correct answer is D. Patients who have had one episode of bleeding from esophageal varices have

an approximately 70% chance of developing a second incident of bleeding, and one third of these

episodes of rebleeding is fatal.

A6

The correct answer is A. Hepatic encephalopathy is seen in end-stage

cirrhosis patients, and can either present or worsen in the presence of

gastrointestinal bleeding. The blood in the upper gastrointestinal tract behaves

essentially as a high protein load, and increases the absorption of ammonia and

nitrogen, which cannot be appropriately metabolized by the liver. GI bleeding

may also predispose for inadequate renal function secondary to hypotension.

Angiotensin I (choice B) is part of the renin-angiotensin-aldosterone system for

blood pressure and sodium ion control.

Calcitonin (choice C) is a hormone secreted by the thyroid, which may be

increased in medullary carcinoma of the thyroid.

Carbon monoxide (choice D) increases in the blood in smokers.

Ceruloplasmin (choice E) is a copper-carrying protein monitored in patients with Wilson disease.

Hemoptysis Case 3

A 53-year-old man consults a physician because he has begun coughing up

sputum tinged with fresh blood. He does not initially report any other symptoms

to his physician. When his physician comments on his hoarseness and cough,

the patient discounts these symptoms and attributes them to his long smoking

history. He cannot say when they began or became worse.

Q 1

The chronic hoarseness suggests dysfunction of which of the following?

/ A. Palate

/ B. Pharynx

/ C. Tongue

Page 66: Abdominal Pain

/ D. Trachea

/ E. Vocal cords

Q 2

Laryngoscopy reveal a fungating tumor of the larynx that is located between the

false and true vocal cords. This tumor is in which of the following sites?

/ A. Aryepiglottic fold

/ B. Infraglottic compartment

/ C. Piriform recess

/ D. Supraglottic compartment

/ E. Ventricle

Q 3

Biopsy of the mass demonstrates a malignancy. Which of the following is the

most likely diagnosis?

/ A. Adenocarcinoma

/ B. Lymphoma

/ C. Oat cell carcinoma

/ D. Sarcoma

/ E. Squamous cell carcinoma

Q 4

In addition to smoking, which of the following is an accepted risk factor for this

patient's tumor?

/ A. Alcohol use

/ B. Cocaine use

/ C. Coffee use

/ D. Marijuana use

/ E. Tea use

Page 67: Abdominal Pain

Q 5

The patient returns to clinic three weeks after receiving the news that his cancer

is inoperable. His wife reports that he has been more withdrawn, eating and

sleeping poorly and "just seems to have lost all hope." On examination, the

patient moves very little, never makes eye contact, and admits to "some"

depressed mood, Ioss of appetite, and sleep disruption. Which of the following

symptoms is of most concern regarding his risk of suicide?

/ A. Decreased appetite

/ B. Decreased energy

/ C. Diminished concentration

/ D. Guilty and worthless feelings

/ E. Hopelessness

Q 6

Some patients present with hoarseness due to a tumor at the lung apex that

involves a nerve that is a branch of which of the following?

/ A. Accessory nerve

/ B. Glossopharyngeal nerve

/ C. Hypoglossal nerve

/ D. Phrenic nerve

/ E. Vagus nerve

Hemoptysis Case 3 Answers

A1

The correct answer is E. Hoarseness specifically suggests dysfunction of the

vocal cords, which produce the sounds that are then articulated to speech with

the pharynx (choice B), palate (choice A), tongue (choice C), teeth, and lips.

While vocal cord dysfunction is specifically suggested by hoarseness, the

anatomic lesion may or may not be present at the level of the vocal cords. Vocal

paralysis may be the result of local tumor or trauma, intracranial lesions

affecting the nucleus ambiguus or its supranuclear tracts, and lesions at the

Page 68: Abdominal Pain

base of the skull, neck, or upper portion of the thorax that involve either the

vagus nerve or the recurrent laryngeal nerves.

Air from the trachea (choice D) passes into the larynx to reach the vocal cords,

but tracheal dysfunction does not produce hoarseness.

A2

The correct answer is E. The interior of the larynx is divided into 3

compartments: the supraglottic compartment (choice D) above the false vocal

cords, the ventricle between the false and true vocal cords, and the infraglottic

compartment (choice B) below the true vocal cords.

The aryepiglottic fold (choice A) is the upper free border of the quadrangular

membrane found in the supraglottic compartment.

The piriform recess (choice C) lies behind the thyroid laminae and the lateral

wall of the supraglottic compartment.

A3

The correct answer is E. The epithelial lining of the larynx is squamous

epithelium, and the vast majority of cancers of the larynx are squamous cell

carcinomas. Squamous cell carcinoma can present with hoarseness, cough,

hemoptysis, or difficulty swallowing. If the symptoms develop insidiously (as in

the case), the patient may not seek medical attention until late in the course.

While very small laryngeal cancers can be treated successfully with surgery

and/or radiation, larger ones are much more problematic, in large part because

complete resection of the cancer and any lymph node metastases can be

difficult to impossible to perform without compromising the many vital neck

structures. Patients treated with partial laryngectomy may retain some speech

ability. Patients treated with total laryngectomy can often learn to speak again

using esophageal speech (gradual belching of air through the

pharyngoesophageal junction), a tracheoesophageal fistula (one way valve

between the trachea and the esophagus which makes a sound when air is

forced across it), or an electrolarynx (sound source held against the neck). With

all three techniques, the sound produced is then turned into articulation by the

patient's pharynx, palate, tongue, teeth, and lips. The other tumors listed in the

choices are very uncommon in the larynx.

Page 69: Abdominal Pain

A4

The correct answer is A. The only two risk factors that you will need to associate

with laryngeal carcinoma are smoking and alcohol use. Cocaine (choice B),

coffee (choice C), and tea (choice E) have not been linked to laryngeal cancer.

Marijuana (choice D) has been suggested as a risk factor for oral cancer in a few

cases, but a causal association has not been established.

A5

The correct answer is E. Hopelessness is an ominous sign and is associated with

a higher risk for suicide.

Decreased appetite (choice A), decreased energy (choice B), and diminished

concentration (choice C) are symptoms indicating the presence of a major

depressive episode, or are the result of the medical illness.

Guilty and worthless feelings (choice D) are symptoms of depression. They do

not predict risk for suicide.

A6

The correct answer is E. The larynx has complex innervation. The vocal cords

and most of the area of the larynx below them are supplied by the recurrent

laryngeal nerve, which is a branch of the vagus nerve that passes beneath the

subclavian artery before returning to the neck to innervate the larynx. From

above the larynx, the superior laryngeal nerve arises from the vagus and divides

into the internal laryngeal nerve and the external laryngeal nerve. The

innervation above the vocal cords is by the internal laryngeal branch of the

vagus nerve. The cricothyroid muscle (the only muscle of the larynx not supplied

by the recurrent laryngeal nerve) is supplied by the external laryngeal branch of

the superior laryngeal, which also branches off the vagus, but which contains

motor fibers originally derived from the accessory nerve (choice A).

The glossopharyngeal nerve (choice B) supplies the pharynx.

The hypoglossal nerve (choice C) supplies the tongue.

The phrenic nerve (choice D) supplies the diaphragm.

Dysphagia case 2

Page 70: Abdominal Pain

A 65-year-old man presents to his physician because he has been having

increasing difficulty swallowing over the past 2 months. He is still able to

swallow liquids, but swallowing solid food now causes severe pain and a sense

of fullness behind his sternum. He has lost 18 pounds

since his swallowing difficulties began. The patient is referred to a

gastroenterologist, who demonstrates a mass lesion of the distal esophagus,

which on biopsy is shown to contain cancer.

Q1

Which of the following is most important in separating the esophagus from the

larynx, and must consequently be carefully passed behind

during endoscopy?

/ A. Arytenoids

/ B. Cricoid cartilage

/ C. Epiglottis

/ D. Pharynx

/ E. Vocal cords

Q2

Which of the following nerves provides the efferent impulses necessary for the

esophageal actions that occur during swallowing?

/ A. Glossopharyngeal

/ B. Hypoglossal

/ C. Spinal accessory

/ D. Trigeminal

/ E. Vagus

Q3

Which of the following approximately represents the proportion of different

esophageal cancer types now being observed in the United States?

/ A. 1/10 adenocarcinoma and 9/10 squamous cell carcinoma

/ B. 1/3 adenocarcinoma and 2/3 squamous cell carcinoma

/ C. 1/2 adenocarcinoma and 1/2 squamous cell carcinoma

Page 71: Abdominal Pain

/ D. 2/3 adenocarcinoma and 1/3 squamous cell carcinoma

/ E. 9/10 adenocarcinoma and 1/10 squamous cell carcinoma

Q 4

Precancerous metaplasia of the esophageal epithelium gives rise to a mucosa

resembling which of the following?

/ A. Mesothelium

/ B. Respiratory epithelium

/ C. Small intestine

/ D. Squamous epithelium

/ E. Stomach

Q5

Frequent use of which of the following has recently been found to probably have

a protective effect against development of esophageal

cancer?

/ A. Acetaminophen

/ B. Alcohol

/ C. Aspirin

/ D. Cigarettes

/ E. Codeine

Q 6

Currently, esophageal cancer has which of the following long-term survival

rates?

/ A. Less than 5%

/ B. 30%

/ C. 50%

/ D. 70%

Page 72: Abdominal Pain

/ E. More than 95%

Dysphagia case 2 answers

A1

The correct answer is C. Endoscopists are very careful when guiding the

endoscope past the epiglottis, which is a pear-shaped portion of elastic cartilage

that can be moved during swallowing to close the larynx, preventing swallowed

material from eventually entering the lungs. The arytenoids (choice A) are the

site of the attachment of the vocal cords (choice E) within the larynx. The cricoid

cartilage (choice B) is in the more distal portion of the larynx.The pharynx

(choice D) is shared by the respiratory and gastrointestinal tracts.

A2

The correct answer is E. The vagus nerve supplies the efferent input into the

esophagus that is necessary for swallowing.

The glossopharyngeal nerve (choice A) provides taste and sensation on the

palate, but the only muscle it supplies is the stylopharyngeus.

The hypoglossal nerve (choice B) moves the tongue during the initiation of

swallowing, but does not innervate the esophagus.

The spinal accessory nerve (choice C) plays no role in swallowing. This nerve

mediates head and shoulder movement and innervates laryngeal muscles.

The trigeminal nerve (choice D) provides general sensation to the mouth and

motor innervation to the muscles of mastication.

A3

The correct answer is C. More recent statistics indicate that the incidence of

adenocarcinoma and squamous cell carcinoma of the esophagus are now

roughly equal. Formerly, approximately 2/3 of the esophageal cancers were

squamous in origin (choice B). Adenocarcinoma of the esophagus is often found

in the distal esophagus.

A4

The correct answer is C. This is an indirect question about Barrett's esophagus,

which is an important precursor of adenocarcinoma of the esophagus. While

Barrett's esophagus was initially defined to be either gastric-type or intestinal-

type metaplasia of the esophagus, more recent studies have shown that the

Page 73: Abdominal Pain

actual problem lesion is more likely to be intestinal metaplasia (diagnosed when

isolated goblet cells are seen in the epithelium) rather than gastric metaplasia

(choice E).

Metaplasia to mesothelium (choice A) or ciliated respiratory epithelium (choice

B) does not usually occur in the esophagus.

The normal epithelium of most of the esophagus is squamous (choice D).

A5

The correct answer is C. An interesting new research observation that may be

exploited in the future is that the incidence of esophageal cancer appears to be

much lower in people who use aspirin frequently.

Cigarettes (choice D) and alcohol use (choice B) have been implicated as risk

factors for esophageal cancer.

Acetaminophen (choice A) and codeine (choice E) have no known effects on the

incidence of esophageal cancer.

A6

The correct answer is A. Esophageal cancer is one of the very bad cancers,

presently with poor long-term survival. The underlying problem is that the

esophagus is only about 3 mm thick, and both metastatic disease and direct

spread (often unresectable) to mediastinal structures is common. Active

research is presently being undertaken to modify this prognosis by using

chemotherapy and radiation therapy prior to surgery, but these modalities have

not yet come into widespread use.

A 55-year-old man with a history of coronary artery disease and alcoholism presents to

the emergency department complaining that he vomited bright red blood twice this

morning. He denies previous episodes of bleeding or abdominal pain. On examination,

he is a malnourished man in acute distress. His blood pressure is 90/50 mm Hg and his

pulse is 110/min. His mucous membranes are dry and his sclera are icteric. Abdominal

examination reveals a distended abdomen with an enlarged, palpable spleen. Purplish

Page 74: Abdominal Pain

striae are seen around the umbilicus. On rectal examination, Iarge hemorrhoids are

seen, but the stool is negative for blood.

Q 1

Which of the following is the most likely diagnosis?

/ A. Erosive gastritis

/ B. Esophageal varices

/ C. Infectious enteritis

/ D. Mallory Weiss tear

/ E. Peptic ulcer disease

Q 2

Which of the following coagulation factors would most likely be unaffected in this

patient?

/ A. Factor ll

/ B. Factor VII

/ C. Factor IX

Page 75: Abdominal Pain

/ D. Factor XIII

/ E. Von Willebrand's factor

Q 3

Which of the following anatomic relationships provides the basis for the patient's

hemorrhoids?

/ A. Coronary vein anastomosis with the esophageal plexus

/ B. Inferior rectal vein anastomosis with the iliac vein

/ C. Paraumbilical vein anastomosis with the inferior epigastric vein

/ D. Superior mesenteric vein anastomosis with the splenic vein

/ E. Superior rectal vein anastomosis with the inferior and middle rectal vein

Q 4

Which of the following structures are found in the portal triad?

/ A. Hepatic vein, common hepatic artery, common bile duct

/ B. Portal vein, celiac artery, common bile duct

/ C. Portal vein, common hepatic artery, common bile duct

Page 76: Abdominal Pain

/ D. Portal vein, falciform ligament, common bile duct

/ E. Portal vein, sinusoids, bile canaliculi

Gastrointestinal Bleeding case 3 answers

A1

The correct answer is B. While all of the answer choices listed must be

considered in the differential, upper gastrointestinal bleeding from esophageal

varices is most likely. This patient displays many of the stigmata of hepatic

disease and portal hypertension: icteric sclera, hemorrhoids, distended umbilical

veins (caput medusae), and a history of alcoholism. In this setting, esophageal

varices would be the most likely. To make this diagnosis definitively, however,

one needs to examine the gastrointestinal tract endoscopically.

Erosive gastritis (choice A) is a source of upper gastrointestinal hemorrhage,

but it seldom bleeds so profusely that the patient becomes hemodynamically

unstable.

Infectious disease in the gastrointestinal tract (choice C) may produce

hemorrhage, but it tends to produce lower GI bleeding.

Mallory Weiss tears (choice D) produce upper GI bleeding. This tearing of the

gastroesophageal junction occurs in alcoholics, but usually a history of retching

precedes bleeding. No such history is elicited here.

Peptic ulcer disease (choice E) can produce brisk upper GI bleeding. It is less likely in this case

because this patient has no history of GI pain.

A2

The correct answer is E. Von Willebrand's factor is a coagulation factor

produced by the vascular endothelium and megakaryocytes. It is the only

protein in the cascade that is not synthesized in the liver. vWF mediates the

adhesion of platelets to the vessel wall basement membrane after vascular

Page 77: Abdominal Pain

injury. Patients with a deficiency of von Willebrand's factor have a tendency to

bleed. It is an autosomal dominant disease, and the ristocetin cofactor activity

test is the best way to clinically assess vWF function.

Factor II (choice A) is produced in the liver. Deficiency is very rare, but can

produce spontaneous or posttraumatic bleeding.

Factor VII (choice B) is produced in the liver.Severe factor VII deficiency is a

very rare cause of bleeding.

Factor IX (choice C) is produced in the liver. A factor IX deficiency is known as

hemophilia B, which is an X-linked disease.

Factor XIII (choice D) is produced in the liver. A deficiency of factor XIII produces delayed

bleeding and poor wound healing.

A3

The correct answer is E. The patient's hemorrhoids are a consequence of his

portal hypertension. The patient has a cirrhotic liver, which impedes circulation

in the portal system. As the pressure rises in the portal system, blood in the

portal circulation begins to backflow into the caval circulation. At the sites at

which the portal system anastomoses with the caval circulation, venous

engorgement occurs. At one such site, the confluence of the superior rectal vein

(portal) with the middle and inferior rectal vein (caval), this venous

engorgement leads to hemorrhoids.

The coronary vein anastomosis with the esophageal venous plexus (choice A)

provides the anatomic basis for the esophageal varices seen in portal

hypertension. As pressure builds in the portal system, venous engorgement

occurs, and varices are produced in the distal esophagus. These varices can be

the site of life-threatening upper GI bleeding.

The anastomosis of the inferior rectal vein with the iliac vein (choice B) is a

caval-caval anastomosis and would not be affected by portal hypertension.

The anastomosis of the paraumbilical vein and the inferior epigastric vein

(choice C) is the portal-caval anastomosis responsible for the purplish striae or

Page 78: Abdominal Pain

caput medusae seen on this patient's abdomen. This circulatory route is an

embryologic remnant, and is only patent when portal pressure rises high enough

to re-open this pathway.

The anastomosis of the superior mesenteric vein and the splenic vein (choice D) marks the origin

of the portal vein. It may have an elevated pressure, but it is not the basis for hemorrhoids.

A4

The correct answer is C. The portal triad contains the portal vein, common

hepatic artery, and common bile duct. It is found in the fold of peritoneum,

called the hepatoduodenal ligament, that separates the greater and lesser

abdominal sacs.

None of the other choices offer a complete answer:

The hepatic vein (choice A) drains the liver into the inferior vena cava (IVC).

The celiac artery (choice B) supplies blood to the anatomic foregut. One of its

branches, the common hepatic artery, travels in the porta hepatis.

The falciform ligament (choice D) is the remnant of the umbilical vein that

passes from the anterior abdominal wall to the superior surface of the liver.

Bile canaliculi (choice E) are microscopic channels that drain bile from the hepatocytes.

A 64-year-old man with a history of coronary artery disease (CAD) comes to the

emergency department with the acute onset of severe, constant, Lower abdominal pain and

rectal bleeding. He reports that he previously has had several episodes of similar, but less severe

pain.

About 12 hours after the onset of pain, the patient began passing copious bright red

blood per rectum. He denies nausea, vomiting, sick contacts, or foreign traveI. Initial

physical examination reveals a distressed man, who is afebrile, but tachypneic, with

scant diffuse abdominal tenderness to palpation. Rectal examination is positive for

blood. Laboratory studies reveal a metabolic acidosis with an elevated serum Iactate.

Q 1

Page 79: Abdominal Pain

Which of the following is the most likely diagnosis?

/ A. Colon carcinoma

/ B. Infectious colitis

/ C. Inflammatory bowel disease

/ D. Ischemic colitis

/ E. Necrotizing enterocolitis

Q 2

The lactate produced from the anaerobic metabolism in the infarcted gut will likely be

which of the following?

/ A. Exhaled as a fruity odor

/ B. Incorporated into glycogen in the liver

/ C. Incorporated into myoglobin in muscle

/ D. Incorporated into urea in the urine

/ E. Secreted by the kidneys unchanged

Q 3

If this patient's disease were drug-induced, which of the following agents would most

likely be responsible?

/ A. Acetaminophen

/ B. Amiodarone

/ C. Cocaine

/ D. Dexamethasone

Page 80: Abdominal Pain

/ E. Nitroglycerin

Q 4

While the patient is in the emergency department, the pain becomes increasingly

severe. Several hours after his initial examination, the patient becomes febrile and is

now exquisitely tender to palpation. He writhes in pain when the physician jostles the

bed. Air is seen under the diaphragm in an upright chest x-ray film. These new findings

suggest which of the following?

/ A. Abdominal aortic aneurysm

/ B. Bowel obstruction

/ C. Cholecystitis

/ D. Hypovolemia

/ E. Perforation with peritonitis

Q 5

Upon surgical exploration of the abdomen, the colon is dull and dusky from the mid

transverse colon to the rectum. The patient has occluded

which of the following vessels?

/ A. Celiac trunk

/ B. Cystic artery

/ C. External iliac artery

/ D. Inferior mesenteric artery

/ E. Superior mesenteric artery

Page 81: Abdominal Pain

______________________________________________________________________

______

Abdominal Pain Case 3 Answers

A1

The correct answer is D. A patient with severe abdominal pain and rectal

bleeding with an unremarkable physical examination is likely suffering from

ischemic colitis. "Pain out-of-proportion to examination" is a classic finding for

ischemic colitis. The previous episodes of less severe pain represent ischemic

angina. An infarction has occurred, as indicated by the rise in serum lactate

secondary to the colon's anaerobic metabolism. The history of coronary artery

disease also suggests this diagnosis, as the atherosclerotic processes that

contribute to his CAD are also likely present in his abdominal vasculature.

Colon cancer (choice A) would produce less acute symptoms, but occasionally,

colon cancer may present acutely with obstructive symptoms. Patients may

have bleeding and abdominal pain, but the pain is typically intermittent and

accompanied by nausea, vomiting, abdominal distention, and absence of flatus.

Infectious colitis (choice B) is incorrect. While patients may have bleeding and

abdominal pain, nothing in the history suggests a disease of infectious origin (no

sick contacts or foreign travel). The acute onset also suggests a vascular event,

rather than an infectious one.

Inflammatory bowel disease (IBD) (choice C) is incorrect because while the

patient reports previous episodes, an elderly man with IBD would likely have a

chronic history of abdominal pain and bleeding.

Necrotizing enterocolitis (choice E) affects premature infants and would not be relevant in this

setting.

A2

The correct answer is B. Lactate is converted into glucose, and then glycogen

in the liver by a process know as the Cori cycle.

Page 82: Abdominal Pain

Choice A is incorrect, as lactate would not be exhaled. A fruity odor on the

breath would be a sign of ketoacidosis.

While some of the carbon from the lactate may be incorporated into peptides

via Krebs intermediates (e.g., choice C), the vast majority would be left as

carbohydrate.

Urea (choice D) represents a means of eliminating nitrogenous waste.

Choice E is wrong, as the kidneys would retain the lactate, rather than excreting it.

A3

The correct answer is C. Cocaine is a sympathomimetic drug that indirectly

acts on both the alpha and beta adrenergic receptors on the vasculature. As

such, cocaine may cause vasospasm in the abdominal vasculature leading to

infarction and ischemic colitis. Similar vasospastic events may occur in the

coronary vasculature, leading to myocardial infarction.

Acetaminophen (choice A) is an analgesic, and would not play a role in

producing ischemic colitis.

Amiodarone (choice B) is an antiarrhythmic, and would not contribute to

ischemic colitis.

Dexamethasone (choice D) is a steroidal anti-inflammatory drug. Not only

would this medication not cause ischemic colitis, it might mask the symptoms

due to its potent anti-inflammatory properties.

Nitroglycerin (choice E) is a venodilator, and would not contribute to ischemic colitis. As a

venodilator, nitroglycerin is used to treat coronary ischemia by reducing cardiac preload.

A4

The correct answer is E. This patient has experienced a bowel perforation. Air

under the diaphragm in an upright chest film provides definitive evidence that a

hollow viscus has ruptured. Air near the liver on a left lateral decubitus (patient

lays with the left side down) is an alternative study to demonstrate perforation.

Page 83: Abdominal Pain

Spillage from the perforated bowel has irritated and inflamed the peritoneum,

resulting in peritonitis. Symptoms of peritonitis include extreme, sharp pain

exacerbated by jostling (patients often report that the bumpy ride to the

emergency department caused extreme pain). Patients will be exquisitely

tender to palpation and percussion and may have abdominal rigidity. Fever

typically accompanies peritonitis.

While an abdominal aortic aneurysm or AAA (choice A) presents as acute

abdominal pain, this pain is described as tearing and may radiate to the back. A

pulsatile abdominal mass may be palpated. The air on the chest film is also

inconsistent with AAA.

This patient does not have bowel obstruction (choice B). Signs and symptoms

of bowel obstruction include: nausea, vomiting, intermittent abdominal pain,

hypovolemia, abdominal distention, absence of flatus, and a "step ladder" bowel

pattern on abdominal films.

Cholecystitis (choice C) typically presents as right upper quadrant (RUQ) pain,

fever, and jaundice. Patients usually have a history of colicky RUQ pain.

While the patient is at risk for hypovolemia (choice D), none of the symptoms listed typify

hypovolemia. Signs and symptoms of mild to moderate hypovolemia include malaise, dry

mouth, thirst, decreased skin turgor, tachycardia, hypotension, and decreased urine output.

A5

The correct answer is D. The inferior mesenteric artery distributes blood to

the embryologic hindgut. This includes the distal 1/3 of the transverse colon to

the rectum. The rectum is spared because it receives circulation from the

inferior rectal artery (not mesenteric).

The celiac trunk (choice A) supplies the embryologic foregut. The first three

branches include the splenic artery, the left gastric artery, and the common

hepatic artery. This patient has no findings in this distribution.

The cystic artery (choice B) supplies the gall bladder. There are no gall bladder

findings in this case.

Page 84: Abdominal Pain

The external iliac artery (choice C) gives rise to the vessels of the lower

extremity. Symptoms of occlusion or stenosis might include buttock and thigh

pain exacerbated by walking. Severe stenosis might give patients buttock and

thigh pain, even at rest.

The superior mesenteric artery (choice E) supplies the embryologic hindgut. This extends from

the duodenum to the proximal 2/3 of the transverse colon

50-year-old man consults a physician because he has developed a chronic, non-productive cough and is experiencing a reduced ability to do strenuous work. His symptoms have developed insidiously. On questioning, he states that he is a smoker and has also worked as acontractor for all of his adult life. Physical examination is notable for the presence of repetitive end-inspiratory basal crackles and finger clubbing. A chest x-ray film shows diffusely distributed, small irregular opacities that are most prominent in the lower lung zones. Localizedareas of pleural thickening are also noted. No large masses are seen.

Q 1

The chest x-ray film is most consistent with which of the following?/ A. Emphysema/ B. Interstitial disease/ C. Lobar pneumonia/ D. Lung cancer/ E. Pleural effusion

Q 2

The patient is sent for spirometry for further evaluation. FEV1 and FVC are both shown to be about 60% of the expected values, and the ratio of FEV1/FVC is 90%. These findings are most consistent with which of the following?/ A. Asthma/ B. Bronchiectasis/ C. Chronic bronchitis/ D. Emphysema/ E. Restrictive lung disease

Q 3The patient's work history is most suggestive of exposure to which of the

Page 85: Abdominal Pain

following?/ A. Asbestos/ B. Beryllium/ C. Coal/ D. Kaolin/ E. Silica

Q 4

In addition to predisposing for pulmonary fibrosis and bronchogenic carcinoma, this patient's disease is associated which of the following?/ A. Basal cell carcinoma/ B. Germ cell tumor/ C. Hemangioma/ D. Mesothelioma/ E. Pleomorphic adenoma

____________________________________________________________________

Cough Case 1 Answers

A1The correct answer is B. Diffusely distributed small irregular opacities suggest the presence of interstitial lung disease.Emphysema (choice A) would produce unusually dark lung fields.Lobar pneumonia (choice C) would produce a "white out" of one or more lung lobes.Lung cancer (choice D), if large, would produce a mass lesion (often involving a bronchus), or, if very small, might not be recognized on chest x-ray.Pleural effusion (choice E) would cause a whitened area due to fluid below the lung

A2The correct answer is E. Spirometry is commonly used to subdivide non-tumorous lung diseases into diseases that are predominately obstructive in nature and those that are predominately restrictive. The process of spirometry involves controlled breathing in and out while airflow is plotted against volume to obtain a continuous loop. FVC (forced vital capacity) is the maximum volume of air that can be forcibly and rapidly exhaled following a maximum inspiration. FEV1 (forced expiratory volume in the first second) is the volume of air expelled in the first second of a forced expiration starting from full inspiration. Restrictive lung disease usually shows a reduction in both FVC and FEV1, and the ratio of

Page 86: Abdominal Pain

FEV1/FVC is greater than 80%. If these readings are obtained in a patient, further studies to measure lung volumes are usually performed to confirm the finding.Asthma (choice A), bronchiectasis (choice B), chronic bronchitis (choice C), and emphysema (choice D) all tend to produce obstructive patterns, with FEV1 disproportionately decreased when compared to FVC.

A3

The correct answer is A. Working history often offers helpful clues about possible toxic exposures that may have contributed to lung disease. Asbestos was formerly a common constituent of insulating material in buildings because of both its insulating properties and its fire-resistant properties. People working on old buildings, particularly when removing the old insulation, are consequently vulnerable to high exposures unless they take care to minimize exposure with respirators. The risk to individuals living and working in old buildings is usually markedly less, since the asbestos is typically found behind walls. Individuals who work in asbestos mines (principally in Canada, South Africa, and the former USSR) may also have high exposures.Beryllium (choice B) is used in the nuclear industry and in x-ray tubes, and was formerly used in ceramics, metallic alloys, and fluorescent lights.Most significant coal (choice C) exposures occur in coal miners.Kaolin (choice D) is a component of clay dust.Silica (choice E) is found in sand and glass, and significant exposures can be seen in individuals working in environments where small particles of these materials may become aerosolized.

A4

The correct answer is D. Asbestos exposure is also linked to late (often after 20 years) development of the rare tumor, malignant mesothelioma. The amphibole forms of asbestos appear to be much more likely to induce mesothelioma than does chrysotile, and some authors have even speculated that the rare cases of mesothelioma in persons with predominant exposure to the serpentine form of asbestos may have been actually related to trace exposures to the amphibole forms. In contrast, all types of asbestos can cause pleural plaques, pulmonary fibrosis, and lung cancer. None of the other lesions listed in the choices have ties to asbestos.Basal cell carcinoma (choice A) is a form of skin cancer.Germ cell tumors (choice B) usually occur in testes and ovary.

Page 87: Abdominal Pain

Hemangioma (choice C) is a benign tumor of blood vessels.Pleomorphic adenoma (choice E) is a salivary gland tumor.

A 14-year-old girl receives a bone marrow transplant as part of her treatment for

acute lymphoblastic lymphoma. During the period of profound

immunosuppression before the marrow engrafts, she develops nonproductive

cough, fever, mild hemoptysis, and pleuritic chest pain. A plain chest x-ray film

shows a pleuraI-based wedge-shaped lesion with focal cavitation. Open chest

lung biopsy reveals necrosis and hemorrhage.

Septate fungal forms with dichotomous 45-degree branching are seen in the

necrotic areas and involving the walls of several blood vessels.

When the fungus is cultured, it is found to be a monomorphic fungus.

Q 1

Which of the following is a monomorphic filamentous fungus?

/ A. Aspergillus

/ B. Blastomyces

/ C. Coccidides

/ D. Histoplasma

/ E. Sporothrix

Q 2

Two commonly encountered invasive fungi are Aspergillus and Candidia. In

tissue specimens, which of the following features is often helpful in

distinguishing these organisms?

/ A. Aspergillus has both hyphae and pseudohyphae

/ B. Aspergillus has budding yeasts

/ C. Aspergillus has germ tubes

/ D. Aspergillus shows dichotomous branching

/ E. Aspergillus shows generally obtuse angles of branching

Q 3

Page 88: Abdominal Pain

Involvement by this patient's infection of which of the following is considered to

have the worst prognosis?

/ A. Brain

/ B. Lung

/ C. Middle ear

/ D. Sinus

/ E. Skin

Q 4

Which of the following is the most appropriate pharmacotherapy for this patient?

/ A. Amphotericin B

/ B. Clotrimazole

/ C. Fluconazole

/ D. Flucytosine

/ E. Griseofulvin

Q 5

Which of the following best describes the mechanism of action of the most

appropriate medication for this patient's disease?

/ A. Forms pores in fungal membranes

/ B. Inhibits the demethylation of lanosterol

/ C. Inhibits squalene epoxidase

/ D. Inhibits thymidylate synthase

/ E. Interferes with the synthesis of ergosterol

Q 6

Toxicity to which of the following organs is most likely to limit the administration

of the most appropriate medication for this patient's disease?

/ A. Brain

/ B. Heart

/ C. Kidney

/ D. Liver

/ E. Lung

Page 89: Abdominal Pain

____________________________________________________________________

Cough Case 5 Answers

A1

The correct answer is A. The fungi that cause "deep infections" in humans are

subdivided into the dimorphic forms (which, depending upon temperature, can

be either yeast forms or hyphal forms) and the monomorphic forms (which grow

in the same general form at different temperatures). Of the fungi listed, only

Aspergillus is monomorphic. Aspergillus is a common saprophytic mold found on

decaying material in the environment throughout the world. It can cause human

diseases, including allergic bronchopulmonary aspergillosis (which is essentially

an allergic reaction to inhaled Aspergillus conidia or spores), fungus ball (in

which the Aspergillus grows without invading in a preexisting cavitary lesion of

the lung), invasive aspergillosis (including pneumonia, meningitis, and other

systemic infections), and cellulitis. Invasive aspergillosis is most often seen in

severely immunocompromised patients with severe neutropenia, notably

including those with a history of transplantation, chronic granulomatous disease,

and leukemia. This patient's presentation is typical for invasive pulmonary

aspergillosis. The other fungi listed (choices B, C, D, and E) are all dimorphic.

A2

The correct answer is D. Aspergillus has hyphae, but not pseudohyphae,

budding yeasts, or germ tubes. The branching is dichotomous and at an acute

(often about 45 degrees) angle (opposite of choice E).

Characteristics to look for with Candida include both pseudohyphae and true

hyphae (choice A), budding yeasts (choice B), and occasionally germ tubes

(choice C, better seen with some culture methods).

A3

The correct answer is A. Cerebral aspergillosis is a feared complication of other

forms of aspergillosis because most patients die despite appropriate antifungal

therapy. Of the other sites listed in the question choices, true invasive infection

of the lung (choice B), as opposed to a fungus ball, is considered the most

serious site of infection, though less so than infection of the brain.

Middle ear and sinus involvement (choices C and D) become clinically worrisome

when dissemination or extension to the brain occurs.

Page 90: Abdominal Pain

Skin involvement (choice E) is worrisome if dissemination occurs.

A4

The correct answer is A. Amphotericin B is the mainstay of therapy of invasive

aspergillosis. This medication has a wide fungicidal spectrum and remains either

the drug of choice or the co-drug of choice for severe infections caused by

Aspergillus, Candida, Cryptococcus, Histoplasma, Mucor, and Sporothrix.

Alternative antifungal agents that are not the preferred therapy but do have

some activity against aspergillus include itraconazole, flucytosine (choice D),

and voriconazole.

Clotrimazole (choice B) is used topically for candidal and dermatophytic

infections.

Fluconazole (choice C) is the drug of choice for esophageal and invasive

candidiasis and coccidioidomycosis, and is used for prophylaxis and suppression

in cryptococcal meningitis.

Flucytosine (choice D) is also synergistic with amphotericin B in candidiasis and

cryptococcosis.

Griseofulvin (choice E) is given orally, but is active only against dermatophytes.

A5

The correct answer is A. The polyene anti-fungal amphotericin B is an

amphoteric compound, with both polar and non-polar structural elements, that

interacts with ergosterol in fungal membranes to form artificial "pores," which

disrupt membrane permeability. Resistant fungal strains are those that appear

to have low ergosterol content in their cell membranes. Amphotericin B is given

by slow IV infusion and penetrates poorly into the central nervous system

(intrathecal injections can be used). It has a half-life of more than 2 weeks and is

removed by both metabolism and renal elimination.

Antifungal agents in the azole class (including ketoconazole, fluconazole, and

itraconazole) interfere with the synthesis of ergosterol (choice E) by inhibiting

the P450-dependent 14-alpha-demethylation of its precursor molecule,

lanosterol (choice B).

Flucytosine is activated by fungal cytosine deaminase to 5-fluorouracil (5-FU),

which can be incorporated into fungal RNA and can be also used to form 5-

fluorodeoxyuridine monophosphate, which in turn inhibits thymidylate synthase

(choice D) and the synthesis of thymine.

Page 91: Abdominal Pain

The anti-dermatophytic drug terbinafine inhibits squalene epoxidase (choice C)

and thus decreases ergosterol synthesis (choice E).

A6

The correct answer is C. Nephrotoxicity is most likely to be dose-limiting, and

may force amphotericin treatment to end prematurely. This is an important

problem, since we have so few drugs with wide spectrum activity against

invasive fungal organisms. An amphotericin B lipid complex (ABLC, Abelcet) that

may be less nephrotoxic in those patients who cannot tolerate conventional

amphotericin is available. Amphotericin B also causes infusion-related reactions

including fever, chills, nausea, vomiting, headache, generalized malaise,

hypotension, and arrhythmias.

Major problems encountered less commonly than renal toxicity include

anaphylaxis, generalized pain, bone marrow toxicity, rash, a variety of cardiac

problems including cardiac arrest (choice B), liver failure (choice D), pulmonary

edema (choice E), and convulsions (choice A).

A 39-year-old woman presents to the emergency department after collapsing at

a party. An interview with her boyfriend indicates that she complained of a

severe headache prior to her collapse. He states that she has no significant past

medical history and takes occasional

vitamin supplements. Her blood pressure is 200/120 mm Hg, pulse is 37/min,

and respirations are 5/min. The patient is unresponsive to commands or painful

stimuli. There is moderate papilledema. The remainder of the examination is

unremarkable. An electrocardiogram

demonstrates normal sinus rhythm without T wave inversions or ST segment

changes.

Q 1

Which of the following is the most likely diagnosis?

/ A. Anterior communicating artery aneurysm rupture

/ B. Atonic seizure

/ C. Cocaine induced myocardial infarction

/ D. Posterior inferior cerebellar artery aneurysm rupture

/ E. Vein of Galen malformation

Page 92: Abdominal Pain

Q 2

Which of the following conditions would predispose this patient to having this

condition?

/ A. Atherosclerosis

/ B. Diabetes

/ C. Hemophilia

/ D. Marfan syndrome

/ E. Protein C deficiency

/ F. Protein S deficiency

Q 3

A CT scan would most likely demonstrate blood in which of the following areas?

/ A. Fourth ventricle

/ B. Lateral ventricles

/ C. Subarachnoid space

/ D. Subdural space

/ E. Superior sagittal sinus

/ F. Third ventricle

Q4

Which of the following drugs could have precipitated this patient's condition?

/ A. Cocaine

/ B. Hashish

/ C. Lysergic acid diethylamide (LSD)

/ D. Morphine

/ E. Pindolol

Q5

Which of the following is more likely to be present in patients with this condition

than in normal persons?

/ A. Early AIzheimer disease

/ B. Fronto-temporal brain atrophy

/ C. Medullary thyroid carcinoma

/ D. Osteosarcoma

/ E. Renal cysts

Page 93: Abdominal Pain

____________________________________________________________________

Headache case 4 answers

A1

The correct answer is A. This patient is presenting with a loss of consciousness,

bradycardia, hypertension, and decreased respirations. While loss of

consciousness has a wide differential diagnosis, the triad of bradycardia,

hypertension, and decreased respirations is known as Cushing's triad, and is

indicative of increased intracranial pressure. The finding of papilledema

confirms that there is increased intracranial pressure. The differential diagnosis

at this point is a spontaneous hemorrhage due to aneurysm rupture, trauma,

vascular malformation rupture, or possibly a massive ischemic stroke. Of the

choices given, anterior communicating artery aneurysm rupture is the most

likely diagnosis. Aneurysms are outpouchings of the arteries of the Circle of

Willis that occur most commonly at the anterior communicating artery, middle

cerebral artery, or posterior communicating artery. They most commonly

present with hemorrhage or headache. In this case, there is likely hemorrhage

and increased intracranial pressure leading to secondary brain herniation.

Treatment of this patient consists of lowering intracranial pressure and treating

the aneurysm surgically.

An atonic seizure (choice B) is a fainting spell in which the patient becomes

hypotonic, but recovers over a short interval. There would be no signs of

increased intracranial pressure.

Myocardial infarction (choice C) from cocaine or other etiology would usually

have electrocardiogram abnormalities and there would be no signs of increased

intracranial pressure.

Posterior inferior cerebellar artery aneurysm rupture (choice D) is a possibility

but these aneurysms are rare, compared to anterior communicating artery

aneurysms.

Vein of Galen malformations (choice E) are a remnant of the fetal circulation

that presents in children as a posterior fossa mass. Actual hemorrhage of these

lesions is relatively rare.

A2

Page 94: Abdominal Pain

The correct answer is D. Connective tissue diseases, such as Marfan syndrome,

weaken blood vessel walls and predispose to aneurysms of any blood vessels in

the body. Marfan syndrome is an autosomal dominant disorder that has been

linked to the FBN1 gene on chromosome 15. FBN1 encodes the protein fibrillin,

which is involved in the formation of elastic fibers found in connective tissue.

Without the structural support provided by fibrillin, many tissues are weakened,

with severe consequences, e.g., aneurysm formation.

Atherosclerosis (choice A) is not thought to be associated with intracranial

aneurysms, which are believed to form from congenitally weak areas at the

junctions of blood vessels. Aortic aneurysms are closely associated with

atherosclerosis.

Diabetes (choice B) is not thought to be associated with intracranial aneurysms.

However, diabetes leads to an increased incidence of atherosclerosis, which

may lead to aortic aneurysms.

The hemophilias (choice C) are blood clotting disorders that do not predispose

patients to aneurysms. These patients bleed profusely from even minor vessel

trauma, however.

Protein C deficiency (choice E) and protein S deficiency (choice F) are blood

clotting disorders leading to thrombosis of arteries and veins. This does not

predispose a patient to aneurysms, however.

A3

The correct answer is C. The subarachnoid space consists of the space between

the pia, which adhere to the brain, and the arachnoid membrane. The circle of

Willis, including the anterior communicating artery, lies in the subarachnoid

space. Subarachnoid hemorrhage is a common presenting symptom of ruptured

intracranial aneurysms. Aneurysmal subarachnoid hemorrhage is usually within

the basilar cisterns, where the circle of Willis lies, while posttraumatic

subarachnoid hemorrhage is usually over the cerebral convexities. Hemorrhage

into the epidural or subdural space is usually secondary to trauma. Epidural

hematomas occur from injury to the middle meningeal artery and subsequent

hematoma formation, and are usually associated with a fracture of the temporal

bone.

Intraventricular hemorrhage (choices A, B, and F) is a much less common

presentation of a ruptured aneurysm. Usually there will be subarachnoid

hemorrhage and intraventricular hemorrhage, rather than isolated

Page 95: Abdominal Pain

intraventricular hemorrhage. Intraventricular hemorrhage often leads to

ependymitis and hydrocephalus from dysregulation of the normal cerebrospinal

fluid production and resorption physiology.

Subdural hematomas (choice D) are usually secondary to trauma, not bleeding

aneurysms. Subdural hematomas form from injury to the bridging veins

between the venous sinuses and the cortical draining veins. Subdurals are

common in elderly patients because they usually have some degree of brain

atrophy and these bridging veins are stretched thin.

There is normally blood present in the superior sagittal sinus (choice E), which

drains the cortical veins from the top of the cerebrum.

A4

The correct answer is A. The key here is to find the drug that leads to

hypertension, and thus is likely to cause an aneurysm to rupture. Cocaine leads

to episodic hypertension due to its sympathomimetic effects. It may be snorted,

smoked, or injected. Cocaine use is associated with cardiac arrhythmia,

myocardial infarction, stroke, and cerebral or aortic aneurysm rupture. Although

not a cause of intracranial aneurysm formation, it may lead to aneurysm

rupture. Cocaine is used for its central effects on dopaminergic neurons, and the

sympathomimetic effects described above are unwanted side effects.

Hashish (choice B) and marijuana contain delta-9- tetrahydrocannabinol (THC),

which is used for its effects on the central nervous system. Other physical

effects include reddening of the eyes, dryness of the mouth and throat,

moderate increase in the heart rate, tightness of the chest (if the drug is

smoked), drowsiness, unsteadiness, and muscular incoordination. Hypertension

is not a common effect of THC.

Lysergic acid diethylamide (LSD) (choice C) is a psychotropic amide with many

poorly-understood central nervous system effects. Significant hypertension does

not generally occur with LSD.

Morphine (choice D) is an opiate analgesic, and would tend to lower blood

pressure, rather than increase it.

Pindolol (choice E) is a nonselective beta-adrenergic receptor blocker. In

addition, pindolol has partial agonist activity, with significantly greater agonist

than antagonist effects at beta-2 receptors. It has negative inotropic and

chronotropic effects and thus is used as an antihypertensive agent. It would help

prevent hypertension.

Page 96: Abdominal Pain

A5

The correct answer is E. Patients with adult polycystic kidney disease have a

much higher incidence of berry aneurysms than the general population.

Hypertension that may accompany the eventual renal failure can contribute to

aneurysm rupture and subarachnoid hemorrhage.

Early Alzheimer-like changes (choice A) are observed in patients with Down

syndrome.

Fronto-temporal brain atrophy (choice B) is seen in Pick disease.

Medullary thyroid carcinoma (choice C) is seen with increased frequency in

multiple endocrine neoplasia (MEN) IIa and IIb.

Osteosarcoma (choice D) is more frequent in patients with familial